• Ebooks & Courses
  • Practice Tests

How to Plan & Write IELTS Problem Solution Essays

IELTS problem solution essays are the most challenging essay type for many people. The way they are worded can vary hugely which can make it difficult to understand how you should answer the question.

Generally, you’ll be asked to write about both the problem, or cause, and the solution to a specific issue. Sometimes, however, you will only be required to write about possible solutions.

The 3 essay types:

  • Problem and solution
  • Cause and solution
  • Just the solution

Hence, it’s essential that you analyse the question carefully, which I’ll show you how to do in this lesson. I’m also going to demonstrate step-by-step how to plan and write IELTS problem solution essays.

Here’s what we’ll be covering:

  • Identifying IELTS problem solution essays 
  • 6 Common mistakes
  • Essay structure
  • How to plan
  • How to write an introduction
  • How to write main body paragraphs
  • How to write a conclusion

Want  to watch and listen to this lesson?

Click on this video.

Click the links to see lessons on each of these Task 2 essay writing topics. 

Once you understand the process, practice on past questions. Take your time at first and gradually speed up until you can plan and write an essay of at least 250 words in the 40 minutes allowed in the exam.

The Question

Here are two typical IELTS problem solution essay questions. They consist of a statement followed by the question or instruction.

1. One problem faced by almost every large city is traffic congestion.

What do you think the causes are? What solutions can you suggest?

2. Since the beginning of the 20th century, the number of endangered species has increased significantly and we have witnessed more mass extinctions in this period than in any other period of time.

State some reasons for this and provide possible solutions.

These are some examples of different ways in which questions can be phrased. The first half of the questions relate to the problem or cause, the second half to the solution.

What issues does this cause and how can they be addressed?

What are some resulting social problems and how can we deal with them?

What problems arise from this and how can they be tackled?

Why is this? How might it be remedied?

What are the reasons for this, and how can the situation be improved?

Why is this happening, and what measures can be taken to tackle this problem?

And here are a few questions where you only have to write about the solution.

How can this situation be improved?

What solutions can you suggest to deal with this problem?

How can this problem be solved?

What measures could be taken to prevent this?

It’s important that you are able to recognise the common synonyms, words and phrases used in problem solution questions. Here are the key words and their synonyms used in the questions above.

  • Problem  – issues, resulting, situation
  • Cause  – reasons, why
  • Solution  – deal with, addressed, tackled, remedied, improved, measures taken, solved, prevent

Before we move on to some common mistakes, I want to quickly explain the difference between a problem and a cause. Read the following examples.

Problem – I've missed the last bus home after visiting my friend for the evening.

Cause – I misread the timetable and thought the bus left at 22.45 when it actually left at 22.35.

The ‘cause’ is the reason for the ‘problem’.  We’ll be looking at question analysis in more detail in a minute.

6 Common Mistakes

These six errors are common in IELTS problem solution essays.

  • Confusing problem and causes questions.
  • Having too many ideas.
  • Not developing your ideas.
  • Not developing both sides of the argument equally.
  • Not linking the problems and solutions.
  • Not being specific enough.

It is common for an essay to consist of a list of problems and solutions without any of them being expanded on or linked to each other. Sometimes, a student will focus on just the problem or only the solution which leads to an unbalanced essay. Both these issues will result in a low score for task achievement.

You must choose just one or two problems and pick solutions directly linked to them. Explain them and give examples.

Another serious error is to write generally about the topic. You need to be very specific with your ideas. Analysing the question properly is essential to avoiding this mistake. I’ll show you how to do this.

Essay Structure

Now let’s look at a simple structure you can use to write IELTS problem solution essays. It’s not the only possible structure but it’s the one I recommend because it’s easy to learn and will enable you to quickly plan and write a high-level essay.

1)  Introduction

  • Paraphrase the question
  • State 1 key problem/cause and related solution

2)  Main body paragraph 1 – Problem or Cause

  • Topic sentence – state the problem or cause
  • Explanation – give detail explaining the problem or cause
  • Example – give an example

3)  Main body paragraph 2 – Solution

  • Topic sentence – state the solution
  • Explanation – give detail explaining the solution

4)  Conclusion

  • Summarise the key points 

This structure will give us a well-balanced essay with 4 paragraphs.

One Problem/Cause & Solution or Two?

Most questions will state problems, causes and solutions in the plural, that is, more than one. However, it is acceptable to write about just one.

This will give you an essay of just over the minimum 250 words. To write about two problems/causes and solutions will require you to write between 350 and 400 words which are a lot to plan and write in the 40 minutes allowed.

It is better to fully develop one problem/cause and solution than ending up with one idea missing an explanation or an example because you run out of time.

The step-by-step essay structure I’m going to show you includes one problem and solution but you can write about two if you feel able to or more comfortable doing so.

How To Plan IELTS Problem Solution Essays

Here’s the question we’re going to be answering in our model essay followed by the 3 steps of the planning process.

One problem faced by almost every large city is traffic congestion.

What do you think the causes are? What solutions can you sugge st?

  • Analyse the question
  • Generate ideas
  • Identify vocabulary

# 1  Analyse the question

This is an essential step in the planning process and will ensure that you answer the question fully. It’s quick and easy to do. You just need to identify 3 different types of words:

1. Topic words

2.  Other keywords

3.  Instruction words

Topics words are the ones that identify the general subject of the question and will be found in the statement part of the question.

One problem faced by almost every large city is traffic congestion .

So, this question is about ‘ traffic congestion’ .

Many people will do this first step of the process and then write about the topic in general. This is a serious mistake and leads to low marks for task achievement.

What we need to do now that we know the general topic, is to understand exactly what aspect of traffic congestion we're being asked to write about.

The  other keywords  in the question tell you the specific topic you must write about. 

By highlighting these words, it’s easy to see that you are being asked to write about the problem of traffic congestion in large cities. Your essay must only include ideas relevant to these ideas.

The instruction words are the question itself. These tell you the type of IELTS problem solution essay you must write. This is a ‘causes and solutions’ question.

# 2  Generate ideas

The next task is to generate some ideas to write about.

There are several different ways to think up ideas. I cover them fully on the  IELTS Essay Planning  page.

We’re going to use the ‘friends technique’. This is the method I prefer as it allows you to take a step back from the stress of the exam situation and think more calmly.

Here’s how it works. Imagine that you are chatting with a friend over a cup of coffee and they ask you this question. What are the first thoughts to come into your head? Plan your essay around these ideas.

Doing this will help you to come up with simple answers in everyday language rather than straining your brain to think of amazing ideas using high-level language, which isn’t necessary.

You might want to try this yourself before reading on for my ideas.

Here are my ideas:

  • Too many cars on the roads – increasing numbers of people own cars, more convenient than buses & trains
  • Inadequate public transport – crowded, old & dirty
  • Poor road layout
  • Rush hour traffic – most people travel to & from work at the same times each day
  • Car sharing, park-and-ride scheme, congestion charge
  • Improve public transport – more frequent and better quality
  • Improve infrastructure – bus lanes, cycle lanes will make it safer for people to cycle
  • Flexible working hours

For each cause you think of, immediately write down a possible solution. This you will ensure that the problems and solutions you think of are linked.

You don’t need to spend long on this as you only need one or two ideas.

I’ve got more far more ideas here than I need as I spent more time thinking about it that I would in the real exam. I’m going to pick just one cause to develop in the essay and one or two solutions.

My advice on making your selection is to choose ideas you can quickly think of an example to illustrate.

Here are my choices:

Cause  – Too many cars on the roads.  Why? – increasing numbers of people own cars, more convenient than buses & trains

Solution  – Park-and-ride schemes

We’re almost ready to start writing our IELTS problem solution essay but first, we have one more task to do.

# 3  Vocabulary

During the planning stage, quickly jot down some vocabulary that comes to mind as you decide which cause and solution you are going to write about, especially synonyms of key words. This will save you having to stop and think of the right language while you’re writing. For example:

  • traffic jam
  • heavy traffic
  • private transport
  • infrastructure

With that done, we can focus on the first paragraph of the essay – the introduction.

How To Write an Introduction

Good  introductions to IELTS problem solution essays have a simple 2 part structure:

  • State 1 key problem/cause and related solution/s (outline sentence)
  • Have 2-3 sentences
  • Be 40-60 words long
  • Take 5 minutes to write

1)  Paraphrase the question

Start your introduction by paraphrasing the question.

Question: One problem faced by almost every large city is traffic congestion.

                  What do you think the causes are? What solutions can you suggest?

Paraphrased question:  

O ne of the most serious issues facing the majority of large urban areas is traffic jams.  

Note my use of synonyms to replace key words in the question statement. You don’t have to replace every key word but do so where possible whilst ensuring that your language sounds natural.

2)  Outline statement

Now we need to add an  outline statement  where we outline the two main points that we’ll cover in the rest of the essay, that is, the cause and the solution I chose earlier. Here they are again.

Cause  – Too many cars on the roads.  Why? – increasing numbers of people own cars, more convenient than buses & trains

And, this is one way to develop them into an outline sentence.

Outline statement:

The main reason for this is that there are too many private cars on the roads these days and a viable solution is to introduce more park-and-ride schemes.

So, let’s bring the two elements of our introduction together.

     Introduction

how to write a problem cause solution essay

This introduction achieves three important functions:

  • It shows the examiner that you understand the question.
  • It acts as a guide to the examiner as to what your essay is about.
  • It also helps to keep you focused and on track as you write.

The two ideas in your introduction will become your two main body paragraphs.

Main body paragraph 1  –  Too many cars on the roads  

Main body paragraph 2  –  Park-and-ride schemes

How To Write Main Body Paragraphs

Main body paragraphs in IELTS problem solution essays should contain 3 things:

  • Topic sentence – outline the main idea
  • Explanation – explain it and g ive more detail

Main Body Paragraph 1

The  topic sentence  summarises the main idea of the paragraph. That’s all it needs to do so it doesn’t have to be complicated.

It plays an important role in ensuring that your ideas flow logically from one to another. It does this by acting as a signpost for what is to come next, that is, what the paragraph will be about.

If you maintain a clear development of ideas throughout your essay, you will get high marks for task achievement and cohesion and coherence.

We’ll now take the idea for our first main body paragraph and create our topic sentence.

Obviously, we’re going to write about the cause of the problem first.

Main body paragraph 1  –  Too many cars on the roads 

Topic sentence:  

The number of people owning cars increases year on year, with most families now having more than one car. 

Next, we must write an  explanation sentence  that develops the idea.

Explanation sentence: 

Most people like the convenience of travelling at the time they want to rather than being restricted to public transport timetables, so they prefer to drive themselves around rather than taking the bus or train. This is despite the fact that they frequently have to sit in long traffic queues as they near the city centre.

Finally, we add an  example  to support our main point. If you can’t think of a real example, it’s fine to make one up, as long as it’s believable. The examiner isn’t going to check your facts. Alternative, you could add another piece of information to support your idea.

Example sentence:

Whenever I have to attend a meeting in the city, I always drive because it means that I can leave home when I want to rather than getting stressed about getting to the station in time to catch the train.

That’s the 3 parts of our first main body paragraph complete. Here’s the finished paragraph.

how to write a problem cause solution essay

We now follow the same process for our second main body paragraph.

Main Body Paragraph 2

Main idea 2  –   Park-and-ride schemes

First, we write the  topic sentence  to summarise the main idea. 

Topic sentence:

A solution that is proving successful in many areas is park-and-ride schemes.

Now for the  explanation sentence  where we expand on this idea.

Explanation sentence:

This is where you park your car for free in a large car park on the outskirts of the city and take a bus for the final part of your journey. The fee you have to pay for the bus trip is usually very small and this public transport system is generally very regular, running every ten minutes or so.

Finally, an  example  to support this point.

A survey carried out in the city of Exeter showed that the rush hour congestion decreased by 10% when the council set up a park-and-ride scheme to the north of the city. There was an additional drop of another 10% in traffic volume when a second scheme began operating to the south.

That’s the 3 parts of our second main body paragraph complete. Here’s the finished paragraph.

how to write a problem cause solution essay

Now we need a conclusion and our IELTS problem solution essay is done.

How To Write a Conclusion

The conclusion is a summary of the main points in your essay and can generally be done in a single sentence. It should never introduce new ideas.

If you're below the minimum 250 words after you’ve written your conclusion, you can add a prediction or recommendation statement.

Our essay is already over the minimum word limit so we don’t need this extra sentence  but you can learn more about how to write a prediction or recommendation statement for IELTS problem solution essays on the Task 2 Conclusions page.

The conclusion is the easiest sentence in the essay to write but one of the most important.

A good conclusion will:

  • Neatly end the essay
  • Link all your ideas together
  • Sum up your argument or opinion
  • Answer the question

If you achieve this, you’ll improve your score for both task achievement and cohesion and coherence which together make up 50% of the overall marks. Without a conclusion, you’ll score below band 6 for task achievement.

You can start almost any final paragraph of an IELTS problem solution essay with the words:

  • In conclusion

        or

  • To conclude

Now all you need to do is briefly summarise the main ideas into one sentence.

Here’s a top tip . Go back and read the introduction to the essay because this is also a summary of the essay. It outlines what you are going to write about.

To create a good conclusion, you simply have to paraphrase the introduction. 

Introduction:

Here is the same information formed into a conclusion.  I’ve also added a personal statement at the end to link back to one of my example sentences. You don’t have to do this but in this case, I think that it rounds the essay off better.

how to write a problem cause solution essay

That’s it. We’ve completed our essay. Here it is with the 4 paragraphs put together.

Finished IELTS problem solution essay.

how to write a problem cause solution essay

Go through this lesson as many times as you need to in order to fully understand it and put in lots of practice writing IELTS problem solution essays from past exam questions. Practice is the only way to improve your skills.

Would you prefer to share this page with others by linking to it?

  • Click on the HTML link code below.
  • Copy and paste it, adding a note of your own, into your blog, a Web page, forums, a blog comment, your Facebook account, or anywhere that someone would find this page valuable.

Like this page?

More help with ielts problem solution essays & other task 2 essays.

IELTS Writing Task 2  – T he format, the 5 question types, the 5 step essay writing strategy & sample questions. All the key information you need to know.

The 5 Types of Task 2 Essay   – How to recognise the 5 different types of Task 2 essays. 15 sample questions to study and a simple planning structure for each essay type.

Understanding Task 2 Questions  – How to quickly and easily analyse and understand IELTS Writing Task 2 questions.

How To Plan a Task 2 Essay  – Discover why essay planning is essential & learn a simple 4 step strategy, the 4 part essay structure & 4 methods of generating ideas.

How To Write a Task 2 Introduction  – Find out why a good introduction is essential. Learn how to write one using a simple 3 part strategy & discover 4 common mistakes to avoid.

How To Write Task 2 Main Body Paragraphs  – Learn the simple 3 part structure for writing great main body paragraphs and also, 3 common mistakes to avoid. 

How To Write Task 2 Conclusions  – Learn the easy way to write the perfect conclusion for a Task 2 essay. Also discover 4 common mistakes to avoid.

Task 2 Marking Criteria  – Find out how to meet the marking criteria in Task 2. See examples of good and poor answers & learn some common mistakes to avoid.

The 5 Task 2 Essay Types:

Step-by-step instructions on how to plan & write high-level essays. Model answers & common mistakes to avoid.

   Opinion Essays

   Discussion Essays

  Problem Solution Essays

  Advantages & Disadvantages Essays

  Double Question Essays

Other Related Pages

IELTS Writing Test  – Understand the format & marking criteria, know what skills are assessed & learn the difference between the Academic & General writing tests.

  • IELTS Writing
  • Problem Solution Essays
  • Back To Top

 * New * Grammar For IELTS Ebooks

how to write a problem cause solution essay

$9.99 each       Full Set   Just   $ 23.97

Find Out More >>

IELTS Courses

how to write a problem cause solution essay

Full details...

how to write a problem cause solution essay

IELTS Writing Ebook

how to write a problem cause solution essay

Discount Offer

$7 each       Full Set Just   $ 21

how to write a problem cause solution essay

Find out more >>

Testimonials

“I am very excited to have found such fabulous and detailed content. I commend your good work.”  Jose M.

“Thanks for the amazing videos. These are ‘to the point’, short videos, beautifully explained with practical examples."  Adari J.

"Hi Jacky, I bought a listening book from you this morning. You know what? I’m 100% satisfied. It’s super helpful. If I’d had the chance to read this book 7 years ago, my job would be very different now."  Loi H.

"Hi Jacky, I recently got my IELTS results and I was pleased to discover that I got an 8.5 score. I'm firmly convinced your website and your videos played a strategic role in my preparation. I was able to improve my writing skills thanks to the effective method you provide. I also only relied on your tips regarding the reading section and I was able to get a 9! Thank you very much." Giano

“After listening to your videos, I knew I had to ditch every other IELTS tutor I'd been listening to. Your explanations are clear and easy to understand. Anyways, I took the test a few weeks ago and my result came back: Speaking 7, listening 9, Reading 8.5 and Writing 7 with an average band score of 8. Thanks, IELTS Jacky." Laide Z.

      Contact

      About Me

      Site Map

      Privacy Policy

      Disclaimer

IELTS changes lives.

Let's work together so it changes yours too.

Copyright  © 2024     IELT Jacky     

All Right Reserved

IELTS is a registered trademark of the University of Cambridge, the British Council, and IDP Education Australia. This site and its owners are not affiliated, approved or endorsed by the University of Cambridge ESOL, the British Council, and IDP Education Australia.

  • How to Order

User Icon

Problem Solution Essay

Nova A.

Problem-Solution Essay - Writing Guide, Definition & Examples!

10 min read

Problem-Solution Essay

People also read

Feeling stuck when it comes to writing a solid problem-solution essay?

You're not alone—many students find it challenging. This essay type requires careful consideration and skillful execution, which isn’t always easy.

But fret not! This guide is your go-to solution. We've got all the crucial steps, structures, and examples to make essay writing a breeze for you.

So, keep reading!

Arrow Down

  • 1. What is a Problem-Solution Essay?
  • 2. Problem Solution Essay Structure
  • 3. How to Write a Problem-Solution Essay?
  • 4. Problem-Solution Essay Examples
  • 5. Problem Solution Essay Topics
  • 6. Problem Solution Essay Checklist

What is a Problem-Solution Essay?

A problem-solution essay is a form of argumentative writing that looks into a specific issue, providing a detailed examination of the problem and proposing effective solutions. 

A problem-solution essay stands apart from narrative , descriptive , and expository essays by focusing sharply on identifying a problem, analyzing its causes, and proposing effective solutions. 

Unlike other types of essays , it places a strong emphasis on presenting practical remedies to real-world problems. This makes it a unique and impactful type of academic essay writing .

Why Write a Problem Solution Essay

There are significant reasons to write a problem solution essay. 

  • First, it encourages the practical application of knowledge by addressing real-world issues and proposing solutions.
  • Develops deeper understanding of societal problems.
  • Refines analytical skills and fosters a solution-oriented mindset.
  • Enhances communication abilities through clear explanation of complex issues and solutions.
  • Facilitates intellectual growth by tackling significant societal challenges.

Order essay

Paper Due? Why Suffer? That's our Job

Problem Solution Essay Structure

In crafting a problem-solution essay, the structure plays a pivotal role in presenting ideas coherently and persuasively. 

Two commonly used structures for this type of essay are the block structure and the chain structure. Each has its unique approach to organizing information, offering writers flexibility in presenting their analysis and proposed solutions.

Block Structure

The block structure, also known as the whole-to-whole or one-side-at-a-time structure, is a straightforward organizational method. 

In this type of structure, the writer dedicates separate blocks of text to first presenting the problem comprehensively. The solutions to the problems are presented in the next sections subsequently. 

This structure allows for a clear separation between the problem and solution sections, providing a systematic and easy-to-follow presentation.

Here is an outline for block structure problem-solution essay:

outlining the problems that will be discussed.


Chain Structure

Contrasting the block structure, the chain structure takes an interconnected approach. 

In this format, the essay addresses a specific problem and promptly proposes a solution. 

The pattern repeats as each problem is introduced, followed immediately by its corresponding solution. This structure aims to maintain a continuous and engaging flow, presenting a transition between problems and solutions. 

Here is a chain structure problem solution essay template:

How to Write a Problem-Solution Essay?

A problem solution essay requires taking a systematic approach. Here are the basic steps for writing this type of essay:

Step 1 - Topic Selection

Choosing the right topic is the first crucial step in writing a problem-solution essay. Pick a real-world issue that genuinely interests you. 

Consider problems that have personal significance or affect your community. The goal is to engage with a problem that allows for meaningful discussion and exploration.

Step 2 - Understanding the Problem

Before diving into solutions, take the time to fully grasp the intricacies of the problem at hand. 

Research the issue thoroughly, aiming to comprehend its various aspects and implications. This step is essential for presenting a well-informed analysis in your essay, ensuring a solid foundation for proposing solutions.

Step 3 - Explore Viable Solutions

Once you've identified and understood the problem, brainstorm possible solutions. 

Think about practical, achievable, and effective ways to address the issue. Consider different angles and perspectives, aiming for solutions that are not only feasible but also have the potential to make a positive impact in real-world scenarios.

Step 4 - Create an Outline

Organize your thoughts and structure your essay by creating a clear outline. 

Allocate specific sections for introducing the problem, proposing solutions, and crafting a conclusion. A well-organized outline serves as a roadmap, guiding you through each step of the writing process.

Step 5 - Write the Introduction

Begin your essay with a captivating introduction . Start with an attention-grabbing essay hook that draws your reader in. 

Clearly state the problem, emphasizing its significance. Conclude the introduction with a concise thesis statement that outlines the solutions you will explore in the essay.

Step 6 - Craft Body Paragraph

Dedicate an individual body paragraph to each problem and its corresponding solution. Elaborate on the details of the problem and present practical solutions. 

Support your ideas with examples, evidence, or real-life scenarios. This section forms the core of your essay, providing a comprehensive exploration of the issues at hand.

Step 7 - Address Possible Objections

Acknowledge and address potential counterarguments to your proposed solutions. 

Anticipating objections adds depth to your essay, showcasing a thorough consideration of alternative viewpoints. By addressing possible objections, you strengthen the overall persuasiveness of your solutions.

Step 8 - Conclude The Essay

Conclude your essay by summarizing the main points without introducing new information. Reinforce the importance of the proposed solutions and leave a lasting impression. 

Consider ending with a call to action or a thought-provoking statement that resonates with your reader. The conclusion should provide a sense of closure to your essay.

Problem-Solution Essay Examples

We know that writing this kind of essay could be a challenge. Here are some problem solution essay samples, you can download and read them for free:

Problem Solution Essay Sample PDF

Financial Problem Solution Essay

Garbage Problem Solution Essay

Problem Solution Essay IELTS

Problem Solution Essay Topics

When picking a topic for your problem-solution essay, think about a few important things. 

Choose something that matters to your audience and is important in society right now. Pick issues that really affect people or communities and need attention. 

Make sure the problem can be realistically solved with practical solutions. Here is a list of problem solution essay questions and topics you can use:

  • Tobacco should be banned to control lung cancer. Discuss.
  • Obesity is caused by genetically modified and processed food. Discuss the solution.
  • Movie scripts should be censored to control violence in youth. Discuss.
  • How to tackle the lack of resources in urban homeless shelters?
  • Government should propose some policies to deal with illegal immigration. Discuss.
  • How can we use social media to improve awareness?
  • Propose some ways to keep your friends and family safe.
  • College students in the United States are overburdened with homework.
  • How to improve the quality of education in high school?
  • Providing equal opportunities to under-privileged children is important. Discuss.

These are some good problem-solution topics that you can tweak to use as your own. 

Problem Solution Essay Checklist

Use this checklist to ensure your problem solution essay is well-crafted and effective:

Clear Problem Definition

Strong Thesis Statement

Thorough Research

Feasible Solutions

Audience Understanding

Logical Structure

Engaging Language

Addressing Multiple Perspectives

Revised and Edited

Careful Proofreading

So, wrapping up, with this guide and checklist, you can now write a problem-solution essay that fulfills its purpose. Just remember to be clear, practical, and interesting. 

But if you still feel unsure or want extra help, our professional essay writing service is here for you. 

Our experts know how to handle any type of essay, making sure it's top-notch. If you need that extra boost, don't hesitate to get in touch with us. We're here to make sure your essay shines and gets the job done.

So, why wait? Place your do my essay cheap request today!

Frequently Asked Questions

What is a problem-solution essay called.

FAQ Icon

A problem-solution essay is often simply called a "problem-solution essay." It's named this way because its primary purpose is to identify a problem and propose effective solutions.

What type of paper is a problem-solution?

A problem-solution essay is categorized as an argumentative essay. It aims to persuade the reader that a particular problem exists and that specific solutions can address it effectively.

What is the problem-solution essay test?

There isn't a standardized test specifically called a "problem-solution essay test." However, in academic settings, instructors may assign problem-solution essays as assignments or assessments to evaluate students' abilities to analyze problems and propose viable solutions.

What is the problem solution essay topics list for IELTS?

Here are ten potential topics for a problem-solution essay suitable for the IELTS exam:

  • Discuss measures to mitigate the impact of climate change on local communities.
  • Analyze strategies to reduce youth unemployment rates globally.
  • Propose solutions to alleviate traffic congestion in urban areas.
  • Suggest ways to combat rising obesity rates among children and adults.
  • Recommend actions to reduce plastic pollution in oceans and waterways.
  • Discuss how to improve access to quality education for marginalized communities.
  • Analyze methods to ensure equitable access to healthcare services in underserved regions.
  • Propose strategies to tackle cyberbullying and its psychological effects on victims.
  • Suggest measures to address water scarcity issues in drought-prone regions.
  • Discuss solutions to make housing more affordable for low-income families in urban areas.

AI Essay Bot

Write Essay Within 60 Seconds!

Nova A.

Nova Allison is a Digital Content Strategist with over eight years of experience. Nova has also worked as a technical and scientific writer. She is majorly involved in developing and reviewing online content plans that engage and resonate with audiences. Nova has a passion for writing that engages and informs her readers.

Get Help

Paper Due? Why Suffer? That’s our Job!

TED IELTS

  • A Beginner’s Guide to IELTS
  • Common Grammar Mistakes [for IELTS Writing Candidates]

Writing Correction Service

  • Free IELTS Resources
  • Practice Speaking Test

Select Page

How to Structure a Cause and Solution Essay [IELTS Writing Task 2]

Posted by David S. Wills | Mar 30, 2021 | IELTS Tips , Writing | 0

How to Structure a Cause and Solution Essay [IELTS Writing Task 2]

Today, we are going to look at the process of writing an answer to a cause and solution IELTS essay . This is a common question type that will generally ask you to describe a cause (or reason) for an issue and then suggest some solutions.

What are Cause and Solution Essays?

In IELTS writing task 2, you may be asked to discuss the cause of a problem and then suggest some solutions to solve it. There are a wide array of possibilities and sometimes the words “cause” and “solution” are not actually used, so it can be a little difficult to spot these.

Here is an example:

Some people get into debt buying things they don’t need and can’t afford. What are the reasons for this behaviour? What action can be taken to prevent people having this problem?

In the first part, it does not say “what is the cause?” Instead, it says “What are the reasons…?” This is why it is important to read carefully and to think in terms of synonyms. In the second question, it does not say “solutions,” but instead says “What actions can… prevent…?” Again, if you read carefully you will realise this means “What are the solutions to the problem?”

Structure for Cause and Solution Essays

Thankfully, it is very easy to structure a cause and solution essay for IELTS. You simply need to write four paragraphs, with one body paragraph about the causes and one body paragraph about the solutions:

  • Introduction – introduce the topic
  • Body paragraph #1 – explain the causes of the problem
  • Body paragraph #2 – explain the solutions to the problem
  • Conclusion – summarise briefly

This is very, very easy to do. However, today I would like to show you a little more. In fact, I am going to discuss some complicated issues to help you produce a more advanced essay structure.

how to structure ielts essays for cause and solution

How Many Causes and How Many Solutions?

When I talk about IELTS essay structures, I often tell people to write just one main idea per paragraph. This is because for IELTS it is really important to give development and if you write too much then it can end up more like a list than an essay.

However, with cause and solution essays, you can get into multiple causes and multiple solutions if you are careful. Whilst it is fine to write one single cause and one single solution, you might find it easier to list many. However, I would suggest that you must structure this more carefully because you need to link the causes and solutions clearly.

For each cause, you could find one direct solution and link them in the following way:

advanced structure for cause and solution essay

I would recommend using a maximum of three problems and solutions for the aforementioned reason of development. If you wrote a list of five or six, you would not realistically be able to explain them properly.

To understand this better, let’s look at an example cause and solution question:

More and more wild animals are on the verge of extinction and others are on the endangered list. What are the reasons for this? What can be done to solve this problem?

When I read this question, I first thought, “Wow! It’s really hard to answer it because it’s such a vast issue!” Thus, I would want to mention many factors. Rather than listing them, I’ll boil them down to three causes and three direct solutions:

CausesSolutions
Destruction of natural habitatsEnding deforestation
Poaching for Chinese medicineBetter education
Commercial fishingLegal limits on trawl fishing

The benefit of this method is that I can include three big ideas about the problems and then counter each of them with a specific solution. The drawback of course is that I cannot go into much detail. This will be more of a problem for the solution section because obviously issues like “ending deforestation” are incredibly complex and require a lot of discussion. However, you cannot say everything for IELTS.

In order to put across the complexity of the situation and show my understanding of it, I will mention several times that it will be difficult to solve and that the problem is very serious. I will conclude my “solutions” paragraph with a note about the unlikeliness of any real change happening because it is true and also in order to counter any complaints about my ideas being unrealistic.

Sample Band 9 Answer

It is apparent nowadays that humans have had a devastating effect on the environment, and in particular we have caused the extinction of countless species of animals. This essay will explore the reasons for this and mention some possible solutions.

To begin with, there are various causes for the reduction in animal populations around the world. Perhaps the main cause is the destruction of their environment. Humans have cut down rainforests and polluted the seas, which has meant that animal no longer have their natural habitat and thus cannot survive. Beyond that, they are poached in order to satisfy the demand for fake medicines in Asia. Every day, elephants and rhinos are killed in Africa and then sent to China. Beyond that, the growing demand for fish has meant that vast fleets of fishing boats trawl the seas, causing the eradication of wildlife there. Many whales and dolphins, for example, are caught up in these nets as a tragic by-product of the industry.

Fixing this problem will be difficult and for many species it is already too late. The most important factor will be the cessation of deforestation in places like Brazil and Malaysia. Humans have to recognise the value of nature rather than focusing on obtaining more land for farming or housing. Education needs to be drastically improved in Asia and also punishments strictly enforced to end the sadistic trafficking of animals for these so-called medicines. Finally, ethical fishing needs to be practised, with limits on trawl nets and ranges for fishing boats. Sadly, none of this is particularly likely due to a lack of concern amongst most of the citizens of the world, and so of course education must be improved before it is too late.

In conclusion, there are various factors that have caused the tragic loss of biodiversity in this world, but there are some steps that could be taken to mitigate the damage.

About The Author

David S. Wills

David S. Wills

David S. Wills is the author of Scientologist! William S. Burroughs and the 'Weird Cult' and the founder/editor of Beatdom literary journal. He lives and works in rural Cambodia and loves to travel. He has worked as an IELTS tutor since 2010, has completed both TEFL and CELTA courses, and has a certificate from Cambridge for Teaching Writing. David has worked in many different countries, and for several years designed a writing course for the University of Worcester. In 2018, he wrote the popular IELTS handbook, Grammar for IELTS Writing and he has since written two other books about IELTS. His other IELTS website is called IELTS Teaching.

Related Posts

How to Improve your IELTS Writing Score

How to Improve your IELTS Writing Score

April 15, 2024

The Best of 2019

The Best of 2019

December 30, 2019

IELTS Listening Section 4 Advice and Practice

IELTS Listening Section 4 Advice and Practice

October 5, 2018

What is a Good IELTS Score?

What is a Good IELTS Score?

October 3, 2022

Leave a reply Cancel reply

Your email address will not be published. Required fields are marked *

This site uses Akismet to reduce spam. Learn how your comment data is processed .

Download my IELTS Books

books about ielts writing

Recent Posts

  • Cambridge IELTS 19 – Sample Answers
  • Commas and Time Phrases
  • Ambition and Success: Sample IELTS Essay
  • Do the Advantages Outweigh the Disadvantages? – Advice About This Question Type
  • Exams vs Continual Assessement [Model Essay]

ielts writing correction service

Recent Comments

  • David S. Wills on Writing Correction Service
  • raquel on Writing Correction Service
  • Lesson Plans
  • Model Essays
  • TED Video Lessons
  • Weekly Roundup

ielts-material

An Ultimate Guide to Writing IELTS Problem Solution Essays

Janice Thompson

Updated On Nov 08, 2023

arrow

Share on Whatsapp

Share on Email

Share on Linkedin

how to write a problem cause solution essay

Table of Contents

Introduction, identifying ielts problem solution essays, how to write a problem solution essay ielts, structure of problem solution essay, common mistakes to avoid in ielts problem solution essays, tips for ielts problem solution essays, problem solution essay ielts topics:.

ielts logo

Limited-Time Offer : Access a FREE 10-Day IELTS Study Plan!

The IELTS problem solution essays are a type of IELTS writing task 2 essay that asks you to discuss a problem and propose possible solutions. These problem solution essays are designed to assess your ability to identify, analyze, and evaluate problems, as well as your ability to develop and articulate effective solutions.

The IELTS problem solution essay topics can be drawn from a variety of sources and are based on real-world situations and issues. Therefore, to do well in writing task 2, it is important to be familiar with the range of IELTS problem solution essay topics.

Each essay type has a unique structure, so it’s important to identify the type of essay you’re writing before you start. Problem solution essays in IELTS are often worded as follows:

It is important to be able to identify the common synonyms, words, and phrases used in problem solution questions. Here are the key words and their synonyms used in the examples above:

  • Problem : issues, resulting, situation
  • Cause : reasons, why
  • Solution : deal with, addressed, tackled, remedied, improved, measures taken, solved, prevent

However, you will mostly be asked to write about both the problem and its solution. The first part of the question will state the problem or cause, and the second part will ask you to identify solutions.

To plan and write a problem solution essay IELTS, you can follow these steps:

  • Understand the question

The first step is to carefully read the question and understand what is being asked. You should identify the problem, the cause(s) of the problem, and the required solution(s).

  • Brainstorm your ideas

Once you understand the question, take some time to brainstorm your ideas. What are the different aspects of the problem? What are the possible causes? What are the different solutions that could be implemented?

  • Organize your ideas

Once you have a good understanding of the problem and its possible solutions, it is time to organize your ideas into a logical structure.

  • Write your essay

When writing your essay, be sure to use clear and concise language. Avoid using complex sentences and jargon. You should also support your claims with evidence from credible sources.

  • Proofread your essay

Once you have finished writing your essay, be sure to proofread it carefully for any errors in grammar, spelling, or punctuation.

Now that we have understood how to write a problem solution essay, let’s have a look at the structure of a problem solution essay.

Enroll in our free  IELTS online coaching  today and learn how to identify and write problem solution essays like a champ!

Let’s do an example problem solution essay to understand the above mentioned structure.

Problem Solution Essay Example with Structure

Introduction:

  • Before you begin writing your problem solution essay,  read  the question and  identify  the problem/ solution.
  • Note down the ideas that come to your mind naturally. For example, look at the table below.
Dumping of industrial wastes into the nearby water bodies. They must be treated, purified and recycled.
Dumping of non-biodegradable wastes in the environment. Stop using plastics.
  • Choose one of the  problems  and discuss it in detail. Here’s an example:
  • Problem : Dumping of industrial wastes into the nearby water bodies.
  • Solution : They must be treated, purified and recycled.
  • Wastewater treatment
  • Biodegradable products
  • Water bodies such as lakes, rivers etc

You should paraphrase the question and outline the problem and solution in your introduction as mentioned below:

Rise in sea water level is one of the serious threats posed on the environment due to global warming. The main reason for this is the letting out of industrial wastes in nearby water bodies and a reliable solution is waste water treatment.

Body Paragraph 1:

The body paragraph 1 must be organised as follows:

  • Main body paragraph 1:  Letting out of industrial wastes in nearby water bodies
  • Central idea:  Industries are increasing in number.
  • Explanation : There are no strict rules in place regarding the environment. So industries let out their wastes into the nearby water bodies such as lakes, rivers, etc which affects the respective eco-system and thereby leading to more serious issues.
  • Example : As installing a wastewater treatment plant is an additional burden and is costly, it is easier to let the waste water into the nearby water bodies as nobody questions this.

The completed main body paragraph 1 will look like this :

Industries are increasing in number. There are no strict rules in place regarding the environment. So industries let out their wastes into the nearby water bodies such as lakes, rivers, etc which affects the respective eco-system and thereby leading to more serious issues. As installing a wastewater treatment plant is an additional burden and is costly, it is easier to let the waste water into the nearby water bodies as nobody questions this.

Body Paragraph 2:

The body paragraph 2 must be organised as follows:

  • Main body paragraph 2:  Industrial wastewater treatment
  • Central idea:  Installing Industrial waste water treatment plants could be beneficial to the environment.
  • Explanation : Industrial wastewater treatment illustrates the processes used for treating wastewater that is produced by industries into a by-product. The treated industrial wastewater may be reused or released to a sanitary sewer.
  • Examples:  There are proven records for reducing water pollution after wastewater treatment.

The completed main body paragraph 2 will look like this :

Installing Industrial waste water treatment plants could be beneficial to the environment and help in reducing global warming effects. Industrial wastewater treatment illustrates the processes used for treating wastewater that is produced by industries into a by-product. The treated industrial wastewater may be reused or released to a sanitary sewer. There are proven records for reducing water pollution after wastewater treatment.

Conclusion:

  • Make sure to sum up all that has been in the previous paragraphs.
  • Use words like in summary, to summarise, to conclude, or as a conclusion, etc.

The final conclusion will look like this:

In summary, every industry should have a wastewater treatment plant installed to treat their waste water which will reduce water pollution and thereby benefit the environment. This will help reduce global warming and ultimately prevent rise in sea water levels.

Therefore, the finished essay will have the following structure:

Rise in sea water level is one of the serious threats posed to the environment due to global warming. The main reason for this is the letting out of industrial wastes in nearby water bodies and a reliable solution is wastewater treatment.

Industries are increasing in number. There are no strict rules in place regarding the environment. So industries let out their wastes into the nearby water bodies such as lakes, rivers, etc which affects the respective eco-system and thereby leads to more serious issues. As installing a wastewater treatment plant is an additional burden and is costly, it is easier to let the wastewater into the nearby water bodies as nobody questions this.

Installing Industrial wastewater treatment plants could be beneficial to the environment and help in reducing global warming effects. Industrial wastewater treatment illustrates the processes used for treating wastewater that is produced by industries into a by-product. The treated industrial wastewater may be reused or released to a sanitary sewer. There are proven records for reducing water pollution after wastewater treatment.

In summary, every industry should have a wastewater treatment plant installed to treat their wastewater which will reduce water pollution and thereby benefit the environment. This will help reduce global warming and ultimately prevent rise in seawater levels.

Looking for IELTS Writing Task 2 preparation books? Get your copy Now!

Following are some of the Common mistakes that should be avoided in IELTS problem solution essays

  • Not understanding the difference between a problem and its causes.
  • Trying to include too many ideas in the essay, without developing them fully.
  • Not considering both sides of the argument equally.
  • Not linking the problems to the solutions.
  • Not being specific enough in the discussion of the problems and solutions.

Read:  All Useful IELTS Writing Lessons & Websites

Here are some of the tips to follow while practicing or writing an IELTS Problem Solution Essays:

  • Read and understand the instructions given in the question (sometimes the question might ask to write about the cause of the issue as well).
  • Follow the word count (no less than 250 words)
  • Plan the problems and solutions you wish to write about, before starting to write.
  • Start the essay with an introduction paragraph and conclude it with a conclusion paragraph, with the body paragraph between the two.
  • If you’re asked to write about both, cause and the solution, then you can write the cause in one body paragraph and the solution in the next body paragraph.

Pro tip:  To avoid a low score in IELTS problem solution essays, focus on one or two problems and identify specific solutions. Explain the problems and solutions in detail, with examples.

Check out some of the  Recent Writing Task 2 Essay Topics for IELTS 2023

Here are some sample IELTS problem solution essay topics:

Get the inside scoop on how to write a high-scoring IELTS problem solution essay from our IELTS expert in our free webinar.  Book your Seat Now !

Also check :

  • IELTS Writing tips
  • IELTS Writing recent actual test
  • IELTS Writing Answer sheet
  • Free IELTS Writing Essay Evaluation and Correction Service
  • IELTS Writing Practice Tests 2023

Frequently Asked Questions

Can I write cause and solution in the same paragraph?

How to write a solution paragraph?

What is the difference between a Problem solution essay and a cause solution essay?

How do you analyze a problem solution essay?

How to identify a problem solution essay?

Practice IELTS Writing Task 2 based on Essay types

ielts img

Start Preparing for IELTS: Get Your 10-Day Study Plan Today!

Janice Thompson

Janice Thompson

Soon after graduating with a Master’s in Literature from Southern Arkansas University, she joined an institute as an English language trainer. She has had innumerous student interactions and has produced a couple of research papers on English language teaching. She soon found that non-native speakers struggled to meet the English language requirements set by foreign universities. It was when she decided to jump ship into IELTS training. From then on, she has been mentoring IELTS aspirants. She joined IELTSMaterial about a year ago, and her contributions have been exceptional. Her essay ideas and vocabulary have taken many students to a band 9.

Explore other Problem Solution Essays

In the Developed World, Average Life Expectancy is Increasing - IELTS Writing Task 2

Post your Comments

Recent articles.

Some People Think That Parents Should Teach Children How to be Good Members of Society Sample Essay

Raajdeep Saha

IELTS Writing Task 2 Sample Essays

Kasturika Samanta

People have more than One Job at the Same Time – IELTS Writing Task 2

Akanksha Tripathi

Ad

IELTSMaterial Master Program

1:1 Live Training with Band 9 Teachers

4.9 ( 3452 Reviews )

Our Offices

Gurgaon city scape, gurgaon bptp.

Step 1 of 3

Great going .

Get a free session from trainer

Have you taken test before?

Please select any option

Email test -->

Please enter Email ID

Mobile Band 9 trainer -->

Please enter phone number

Application

Please select any one

Already Registered?

Select a date

Please select a date

Select a time (IST Time Zone)

Please select a time

Mark Your Calendar: Free Session with Expert on

Which exam are you preparing?

Great Going!

  • Skip to primary navigation
  • Skip to main content
  • Skip to primary sidebar
  • Skip to footer

IELTS Advantage

IELTS Advantage

IELTS Preparation Courses

IELTS Writing Task 2 Problem and Solution Essay Lesson

how to write a problem cause solution essay

Problem Solution Essay

This lesson on how to write a problem solution essay will:

  • discuss common mistakes;
  • show you how to analyse the question;
  • show you how to think of ideas;
  • give you a structure that can be used again and again on all problem solution IELTS essays;
  • describe how to write an introduction , main body paragraphs and conclusion; and
  • give you a full band 9 sample answer.

Problem/solution questions are one of the most common IELTS Writing Task 2 questions on the academic paper. Despite being very common, many students fail to do well in these questions. This post will look at some of the most common mistakes and then take you through how to answer these questions step-by-step.

how to write a problem cause solution essay

Common Mistakes

  • The most common mistake for problem solution essays is not expanding on your ideas and instead simply listing lots of problems and solutions. The examiner does not want a list of all the problems and solutions you can think of, and please don’t do this in the exam. Instead, if you look at how the exam is marked , the examiner wants you to pick one or two problems and solutions and then expand on them with explanations and examples. More on how to do this below.
  • Another common mistake is writing about problems and solutions that are not directly linked to the question. You should be like a sniper when answering the question and only give very specific ideas rather than ideas that generally talk about the overall issue. This has a lot to do with how you identify keywords and micro-keywords in the questions which we will look at below.
  • Lots of people think of good ideas for problems and then fail to link their solutions to these problems. Each problem should have a solution directly linked to it, or in other words, it should solve the actual problem.
  • Finally, some candidates think of really good problems and solutions that answer the question properly and then expand their answers with explanations and examples, but they talk too generally. Instead, you should be thinking of specific examples and explanations. We will look at how to avoid this below.

Analysing the Question 

This is one of the most crucial parts of answering any IELTS writing question. If you don’t take the time to think properly about what the examiner is asking you to do, then it is very difficult to answer the question correctly.

We analyse the question by thinking about three things:

  • micro-keywords
  • action words

Keywords are the words that tell us what the general topic is.

Micro-keywords identify which part of the general topic the examiner wants you to discuss. They often give an opinion, qualify the statement or talk about a sub-category of the bigger general topic.

Action words tell us what the examiner wants us to do.

Problem Solution Sample Essay

Global warming is one of the biggest threats humans face in the 21st Century, and sea levels continue to rise at alarming rates. 

What problems are associated with this, and what are some possible solutions? 

If we look at this question, we can see that the keywords are ‘ global warming ‘. This is our general topic. We will write about this, but we cannot write about any problems associated with global warming. If we do this, we have not answered the question properly. We, therefore, need to look at the micro-keywords.

The micro-keywords are ‘ humans ‘ and ‘ sea level rise ‘. So instead of writing just about the huge topic of global warming and any problems associated with that (such as increased storms, extinction of certain animals, erosion of soil), we have to talk about how particularly sea level rises will affect humans . If we talked about the problems affecting the ‘planet’ or ‘animals’ or the ‘atmosphere’, we would not be answering the question.

The action words are problems and solutions .  Our task is, therefore, to write about that and only that. It does not ask our opinion about the disadvantages, advantages, or causes, just the problems and solutions. If we discussed the causes of sea level rise, we would not be answering the question.

For more information, go to effectively analyse an IELTS question .

How to Think of Ideas 

how to write a problem cause solution essay

Now that we know exactly what the question is asking us to do, we need to think of specific and relevant ideas. There are many strategies for thinking of ideas for IELTS task 2 questions  TO THINK OF IDEAS FOR IELTS WRITING TASK 2  but for problem-solving questions; I like to use something called the ‘coffee shop method’.

Instead of brainstorming or mind-mapping- which take too much time and lead to irrelevant ideas in my opinion- you should pretend you are in a coffee shop with a friend and they have just asked you a simple question. In this case, it would be “What are the problems and solutions associated with sea level rise on humans?”

If you were talking to a friend about this, I’m sure you would have no problem thinking of at least 2 or 3 problems and solutions. This method takes you out of an exam situation and puts your mind into a more relaxed environment. Try it and see. If you don’t like it, try one of my other methods.

There are several problems and solutions, including:

Problem : flooding of people’s homes and businesses

Solution : build flood barriers or move to higher areas

Problem : loss of agricultural land and starvation

Solution : switch to more suitable crops

Problem : displacement of millions of people

Solution : move people in a planned and orderly way before the floods

Problem : groundwater undrinkable

Solution : build desalination plants

As you can see, I didn’t think of lots of problems and then lots of solutions. For each problem, you should think of a solution that directly solves this problem.

You now have lots of ideas, but now you must decide which ones to use. I always tell my students to pick the ones they know most about, i.e. that they can explain and give relevant examples.

how to write a problem cause solution essay

I advise my students to use a basic four-paragraph structure with all problem solution IELTS essays. Your four paragraphs should look something like this:

Paragraph 1- Introduction

Paragraph 2- Problems

Paragraph 3- Solutions

Paragraph 4- Conclusion

At a sentence level, your structure should look like this:

Introduction 

1- Paraphrase question

2- Outline sentence

3- State problems

4- Explain first problem

5- Explain second problem

6- Example of second problem

7- State solutions

8- Explain solution to first problem

9- Explain solution to second problem

10- Example of solution to second problem

Conclusion 

Sentence 11- Summary of main points in paragraphs 2 and 3

For more structures, check out our IELTS task 2 structures guide .

Now let’s look at each paragraph in more detail.

how to write a problem cause solution essay

The introduction will have two sentences: a paraphrase of the question and an outline statement.

Paraphrasing is simply saying the sentence again with different words but with the same meaning. We can do this by using synonyms and/or changing the order of the words.

Question-  Global warming is one of the biggest threats humans face in the 21st Century, and sea levels continue to rise at alarming rates. 

Paraphrased- Climate change is among the principal dangers facing people this century, and ocean levels are increasing dramatically.

As you can see above, I have used synonyms to change the words of the questions, but it still has the same meaning. The examiner will look for your ability to do this in the exam, so practising this skill is a good idea.

Our outline sentence is next, which tells the examiner what they will read in the rest of the essay. This makes it very clear to the examiner and makes the rest of the essay much easier to understand. You will, therefore, gain marks for coherence and cohesion.

Our outline sentence should look something like this:

This essay will first suggest that the biggest problems caused by this phenomenon are the loss of land and the flooding of homes and then argue that pollution reduction and building flood protection are the most viable solutions.

Our introduction will, therefore, look like this:

Climate change is among the principal dangers facing people this century, and ocean levels are increasing dramatically. This essay will first suggest that the biggest problems caused by this phenomenon are the loss of land and the flooding of homes and then argue that pollution reduction and building flood protection are the most viable solutions.

It should be noted that this introduction does not contain a thesis statement. This is because this particular question does not ask us for our opinion. However, IELTS problem solution questions sometimes do ask you for your opinion, and you should then include a thesis statement.

Problems Paragraph 

Our problems paragraph will have this structure:

Sentence 1- State problems

Sentence 2- Explain first problem

Sentence 3- Explain second problem

Sentence 4- Example of second problem

State problems : The foremost problems caused by climbing sea levels are that land is being lost and peoples’ residences are often flooded.

Now that we have stated the problems, we must explain these. You should always consider your audience to be someone with no specialist knowledge in this area, and you, therefore, need to explain what everything means. Don’t assume that the IELTS examiner is educated and knows what you are talking about. These assumptions will stop you from writing what you need.

Explain first problem : As water levels rise, low-lying land is submerged, and many countries become smaller.

Explain second problem : Furthermore, millions of people worldwide live in coastal areas, and if the sea rises by even a few feet, they are inundated with water and lose their property.

Now we must give an example of what we are talking about. When we give an example, it should be as specific as possible.

An example of a very general example would be:

Lots of people in the world have experienced floods recently. 

This is far too general to be considered a good example.

Example : The devastation brought about by this was clear for all to see during the 2011 Tsunami in Japan, in which millions of people were displaced.

This example is much more specific. Stating a place and/or date can help you make your examples more specific.

Our second paragraph will look like this:

The foremost problems caused by climbing sea levels are that land is being lost and people’s residences are often flooded. As water levels rise, low-lying land is submerged, and many countries become smaller. Furthermore, millions of people worldwide live in coastal areas, and if the sea rises by even a few feet, they are inundated with water and lose their property. This devastation was clear for all to see during the 2011 Tsunami in Japan, in which millions of people were displaced.

Now we must move on to our solutions.

Solutions Paragraph

Our solutions paragraph will have this structure:

Sentence 1- State solutions

Sentence 2- Explain solution to first problem

Sentence 3- Explain solution to second problem

Sentence 4- Example of solution to second problem

State solutions : Possible solutions to these problems would be to reduce the amount of pollution created and build flood barriers.

We now need to explain how our solution will help solve the problem. Again, do not assume that the examiner has any specialist knowledge of this topic, so you need to explain what you mean.

Explain first solution: If each person reduces their carbon footprint, the negative effects on the environment will be reduced, which will mean that the water level will stop rising.

Explain second solution : Furthermore, flood defences, such as dikes, dams, and floodgates, could be built along coasts and waterways, thereby stopping the water from reaching populated areas.

Example : The Netherlands is one of the most populated areas in the world and one of the most vulnerable to flooding. They have successfully employed various flood defence systems.

Our whole solutions paragraph will look like this:

Possible solutions to these problems would be to reduce the amount of pollution being created and to build flood barriers. If each person reduces their carbon footprint, the negative effects on the environment will be reduced, which will mean that the water level will stop rising. Furthermore, flood defences, such as dikes, dams, and floodgates, could be built along coasts and waterways, thereby stopping the water from reaching populated areas. The Netherlands is one of the most populated areas in the world and also one of the most vulnerable to flooding, and they have successfully employed various flood defence systems.

We have now answered the question and need to sum up what we have said in the conclusion.

how to write a problem cause solution essay

The conclusion should have no new ideas but instead should list the main points from the previous two paragraphs. You can also use synonyms in this paragraph to avoid repetition.

Conclusion : To conclude, stemming the rising tides caused by increasing global temperatures is one of the foremost challenges we face, and it will ultimately lead to some countries losing landmass and many of the world’s cities being left underwater, but possible solutions could be to protect our environment and to utilise the flood prevention techniques already used by countries like Holland.

Our whole conclusion for this problem solution essay will look like this:

To conclude, stemming the rising tides caused by increasing global temperatures is one of the foremost challenges we face, and it will ultimately lead to some countries losing landmass and many of the world’s cities being left underwater, but possible solutions could be to protect our environment and to utilise the flood prevention techniques already used by countries like Holland.

Problem and Solution Sample Essay

Here is the whole essay:

Climate change is among the principal dangers facing people this century, and ocean levels are increasing dramatically. This essay will first suggest that the biggest problems caused by this phenomenon are the loss of land and the flooding of homes and then argue that pollution reduction and building flood protection are the most viable solutions. The foremost problems caused by climbing sea levels are that land is being lost and peoples’ residences are often flooded. As water levels rise, low-lying land is submerged and many countries become smaller. Furthermore, millions of people all over the world live in coastal areas, and if the sea rises by even a few feet, they are inundated with water and lose their property. The devastation brought about by this was clear for all to see during the 2011 Tsunami in Japan, in which millions of people were displaced. Possible solutions to these problems would be to reduce the amount of pollution being created and to build flood barriers. If each person reduces their carbon footprint, the negative effects on the environment will be reduced and this will mean that the water level will stop rising. Furthermore, flood defences, such as dikes, dams, and floodgates, could be built along coasts and waterways, thereby stopping the water reaching populated areas. The Netherlands is one of the most populated areas in the world and also one of the most vulnerable to flooding and they have successfully employed various flood defence systems. To conclude, stemming the rising tides caused by increasing global temperatures is one of the foremost challenges we face and it will ultimately lead to some countries losing landmass and many of the worlds’ cities being left underwater, but possible solutions could be to protect our environment and to utilise the flood prevention techniques already used by countries like Holland.

I hope this post helps you with IELTS problem solution essays, and if you have any questions, please comment below.

how to write a problem cause solution essay

Next Steps 

If you found this lesson useful and it has helped you write a problem solution essay, you should also check out our lessons on task 2 opinions essays , discussion essays and advantages and disadvantages essays .

Do you need me to correct your essays and give you feedback on them? Check out our essay correction service .

The best way to keep up to date with posts like this is to like us on Facebook. There are also lots of practice activities for you to do on the Facebook page.

' src=

About Christopher Pell

My name is Christopher Pell and I'm the Managing Director of IELTS Advantage.

I started IELTS Advantage as a simple blog to help 16 students in my class. Several years later, I am very humbled that my VIP Course has been able to help thousands of people around the world to score a Band 7+ in their IELTS tests.

If you need my help with your IELTS preparation, you can send me an email using the contact us page.

The Definitive Guide to Writing a Problem Solution Essay

06 February, 2021

13 minutes read

Author:  Josh Carlyle

In this article, we cover the basics of problem solution essay writing. We will explain what a problem and solution essay is in academic and straightforward terms. We shall also cover the four essential components that make this essay coherent. With these four components in mind, we will offer guidance on the outline structure and provide some general writing tips on research and problem solving, as well as some topics and essay samples.

Problem Solution Essay

What is a Problem Solution Essay?

A problem solution essay is a staple of humanities and social science subjects. These essays cover a pressing issue, examine how it causes problems, and offer solutions to these problems. The topic for problem solution essay papers can be incredibly diverse. The problem could be local, regional, or global. It could affect a wide range of people or be part of the discourse on an arcane and obscure aspect of computing.

problem solution essay

If you are wondering how to write a problem solution essay, look no further than its name. The core of a problem solution essay is in the title. In this type of essay, a problem is presented, and a solution is offered. Doing this well requires presenting the issue in an audience-appropriate way and then offering solutions that thoroughly negate any critiques of those solutions. 

4 Components of a Problem Solution Essay

In a problem and solution essay, the following aspects must be included:  

  • The Situation: this opening part is where the foundation of the problem lies. It is not an introduction in the sense that you may be familiar with or an abstract that covers the entirety of your thesis. Rather, it is a short and straightforward briefing that will make a layperson familiar with the situation at hand.
  • The Problem: this part provides specifics of the problem. Detailed dissections of evidence take place here; we’ll summarise those later in the evaluation section. 
  • The Solution: this component covers push-back specific solutions you may encounter. Part of a correct answer is thinking of your opponents’ perspectives and ensuring that they address their assumptions and points.
  • The Evaluation: in this section, it is crucial to write with brevity and wit; this will make your position memorable. Readers will often remember the last part of essays they read, so make sure it represents a microcosm of your essay as a whole. 

These components are the essence of the structure required in a problem solution essay. The actual structure you will work with will require a finer granulation of sections. For instance, in the solution section, the critiques will also need to have a review applied, demonstrating their lack of applicability or irrelevance. 

Problem Solution Essay Outline

The best guide for a problem solution essay outline is the four components mentioned above. Cover the situation, the problem, the solution, and then evaluate both or all sides of the story. 

To drill down further into the outline, you should have an introduction that will set out your paper’s structure. Then you should present the situation. Keep this section free from emotive language. Use it to ensure the reader has all the facts, and imagine that you want everyone reading the paper to be on a level playing field in terms of knowledge and framing of the problem.

In the problem section, you must explain why there is a particular issue. At this point in your essay, ensure that you do not leave the door open to other causes of the problem. Find ways to make the problem something that the reader cares about and wants to solve, but beware of assuming the reader is on your side simply because you’ve said some things and made some points. 

Once the situation has been explained, and the problem elucidated, present your solution. It  should use evidence, and you should be able to explain how it directly relates to the question.  Don’t use ad hominem attacks or go polemic. Unravel the problem with your solution. Take it to step by step and keep your writing composed.

In the evaluation section, you should find your argument’s weaknesses and the views that find fault with your solution.

How To Find Solutions to Your Problems

You can find solutions to your problems by researching them; someone will have asked the question before, or one very similar to it. You can talk to your peers and even conduct polls on social media to gauge the public’s position on various solutions. 

Another way of finding solutions include flipping your perspective. Take the position of the other side and see the world through their gaze. By putting yourself in the opponent’s shoes, you can see weaknesses in your ideas and perhaps adjust them to take on more relevance to more people or factor in an aspect that you had overlooked.

Ultimately, the best way to find solutions is to read and read some more. Try looking in your library if you prefer books to the internet, but don’t leave Google out of it. Using the search engine correctly, you can dig up all kinds of books, papers, and presentations that will be very useful in your studies.

Problem Solution Essay Topics

The list of problem solution essay topics is very long. As mentioned earlier, the problem can be a local issue, affecting a specific demographic or being universal. Regardless of the topic you choose, there are another million waiting for an answer.

To start you off, the climate crisis is an excellent area for debate. 

  • How do we deal with stranded assets in the fossil fuel industry and financial sector? 
  • What should the laws be surrounding new cars run on petrol and diesel? 
  • How should electrification be carried out in developing nations? Is a carbon tax a viable way to reduce emissions? 

Other problem solution essay ideas might include social media’s effect on dopamine levels, gerrymandering in electoral districts, or the working week’s length. 

Problem Solution Essay Examples

Reading previous papers is a fantastic method to improve your grades. Study essays that get top marks and some of the ones which get lower marks. The difference will be apparent in their vocabulary, logic, and evidence. Read a few problem solution essay sample papers but don’t plagiarize them; always cite your sources. Here are a few examples to help you get started: 

https://www.eapfoundation.com/writing/essays/problemsolution/  

https://www.ieltsbuddy.com/problem-solution-essays.html  

Writing Tips for a Problem Solution Essay

An essay is a complex task to complete. That’s why it’s useful to break down the whole into several steps. 

Step one is to conduct some thorough research and planning. If you have free choice over the problem at hand, then start by brainstorming some groups you belong to and decide whether they’re officially mandated ones like a sports team or a club, or broader, like your sex, gender, nationality, or language grouping.

After this, jot down some issues in the groups. Are you struggling to manage the payment plan for your sports hall? Has there been an instance of bullying or shaming in your school? With the topic in mind, move onto research. 

When researching your topic, it’s always useful to remember that there is nothing new under the sun. It is almost definite that similar problems have arisen before, and most solutions already exist.

Then, conduct and compose a literature review on the topic. A literature review is an excellent addition to your essay as a standalone section. It helps to situate your issue within the world and builds relations to other similar problems. 

You may think you have nothing to say about the problem or find it hard to justify your opinion. Well, in the problem solution essay, your voice matters. Always try to back up what you’re saying through evidence and try not to stray into writing a polemic. Yet, passion and emotion are useful tools for framing the problem. Just try not to make these responses the whole essay.

As far as solving problems goes, you can use some general mechanics to come up with solutions. The following paragraphs will present some of these logical mechanics; feel free to use them in your writing:

  • Add something new: this solution works when something is lacking. The problem may be a lack of funding, equipment, or motivation. The assumed problem when using this angle of attack is that there is a lack of resources available. Concrete examples of this could include more teaching assistants in education, longer opening hours at a library, or more significant legislation to protect the environment.
  • Remove something: this is the inverse of the previous point. Inversing is a useful strategy for thinking about problems in general. If something is too crowded or too busy, it might be a solution to limit exposure to something and devise a solution. 
  • Education: learning is a more specific aspect of ‘adding something’; it presupposes that a lack of information and awareness is the cause of the problem. If people had this knowledge, the theory goes that the issue would disappear or reduce because people could make informed decisions and correct their behavior.
  • Enforcement: if something like school rules, or even the law, is being ignored, proper enforcement might be a solution. Enforcement has its own sets of problems. This angle is an excellent way to write an unfolding and varied essay as it requires lots of discussion around proper enforcement. Making people do things they don’t want to do is a tricky situation and is riddled with structural and psychological issues.
  • Compromising: proposing compromise through mediation or bi-partisan effort is another complex solution. To work effectively, it has to involve people who possess robust negotiation techniques. But settlements happen all the time, so they’re a powerful solution to many problems. It may be useful to learn about zero-sum and relative issues to argue this case correctly.
  • A change in leadership: leaders can become stale and cumbersome; they may get weighed down by responsibility and have a low tolerance for change. Although, many problems require systemic changes, such as the climate crisis. With an established leader in charge, progress could be slow as they may be blinkered by their position. In this case, they are proposing an election or vote of confidence as a solution that can break the deadlock and offer people a chance to voice their concerns through voting.

One could surmise that the problem and solution essay is an incredibly relevant style of writing. By dissecting an issue and coming up with solutions, you learn a skill that is useful in many careers and practices. But writing one effectively requires both passion and perseverance; writing about topics that move us, though letting the fire burn too brightly, can put readers off or lead them to ignore their blind spots. 

Follow the structures set out above and make sure to proofread your essays before submission. Finding a good editor is always a positive step; they can help to rephrase your words so that your argument comes across more fluently. 

Writing drafts is good practice, although not always possible due to time constraints. Ideally, you should work through two drafts before submitting a final piece; if the essay makes up a small part of your overall grade, adjust the drafting process accordingly. 

Write a Problem Solution Essay with HandmadeWriting

Problem solution writing has been speaking truth to power for millennia. HandmadeWriting loves seeking solutions as much as it loves a simple academic essay or lab report. It’s hard to become a good essay writer without getting critical at times. That’s why we pride ourselves on producing some of the most compelling content around. So be the next one to enjoy our writing and get an A+ for it.

A life lesson in Romeo and Juliet taught by death

A life lesson in Romeo and Juliet taught by death

Due to human nature, we draw conclusions only when life gives us a lesson since the experience of others is not so effective and powerful. Therefore, when analyzing and sorting out common problems we face, we may trace a parallel with well-known book characters or real historical figures. Moreover, we often compare our situations with […]

Ethical Research Paper Topics

Ethical Research Paper Topics

Writing a research paper on ethics is not an easy task, especially if you do not possess excellent writing skills and do not like to contemplate controversial questions. But an ethics course is obligatory in all higher education institutions, and students have to look for a way out and be creative. When you find an […]

Art Research Paper Topics

Art Research Paper Topics

Students obtaining degrees in fine art and art & design programs most commonly need to write a paper on art topics. However, this subject is becoming more popular in educational institutions for expanding students’ horizons. Thus, both groups of receivers of education: those who are into arts and those who only get acquainted with art […]

Preparation for the IELTS Exam

Two easily confused essays (problem / causes solution essay)

What is the difference between a causes solution and a problem solution essay.

Problem solutions and causes solutions essay are very similar but there is a subtle difference. One type asks about the problems and the other type asks about the causes .

It is very important to spend a couple of minutes analysing the task question so that you know what to write about in the essay. I have seen many good essays lose marks in Task Response because the writer did not find the issues in the task question or did not fully understand what to write about.

In both types of essay, you have to mention the problem but in a causes solution essay you are mainly focused on the cause of the problem and that is what you will write about as well as the possible solutions.

What are the differences between these two essay questions below?

how to write a problem cause solution essay

Analysing the question

The first essay is asking in the instructions words: What problems does this cause? The word ‘this’ refers to the issue of ‘professionals leaving their own poorer countries to work in developed countries’ 

 So you have to write about: the problems that happen as a result of these professional people (doctors, teachers, etc) leaving their own developing countries. You then have to write about: possible solutions to this issue of these professional people wanting to leave their own countries.

In the second essay, the instruction words ask: What are the causes of this problem? . The word ‘this’ again refers to the issue of ’professionals leaving their own poorer countries to work in developed countries’ so it’s the same issue as in essay 1. 

However, you have to write about why this problem is happening in the first place . In the essay, you do not need to focus on what happens when these professionals leave their countries , instead, you will write about what is causing this issue to happen. The solutions paragraph will be the same as the first essay, addressing ways to solve the cause of the problem.

Of course, you can mention the result of people leaving their own countries and the problems it will cause but remember that this essay is asking you what are the causes of the problem.

This is the reason why I advise taking a couple of minutes to really be sure you understand what the task question is asking you to write about. The second essay task would be considered a ’causes solutions’ essay.

Other instruction words to indicate a causes solutions essay:

  • What are the reasons for this happening?
  • Why is this happening?
  • What is the cause of this issue?

IELTS likes to paraphrase the instruction words in essay tasks, so you may see the above instruction words in this kind of essay, they are all asking about the causes.

Task Response

If you do not analyse the question well you could end up writing a different essay and get a much lower score in Task Response.

What does it say in the IELTS marking criteria for task response?

For Band 5 it says: addresses the task only partially… For Band 6 it says: addresses all parts of the task although some parts may be more fully covered than others.

 For Band 7 it says: addresses all parts of the task.

If you want a band 7 make sure you analyse the questions well, plan your main ideas and supporting points and make sure you explain your points with a specific example. Do not give too many examples like a list.

In fact, you can write about one problem and one solution (or one cause and one solution) as long as you develop your answer well.

Getting ideas and planning

I always advise taking at least 10 minutes to plan before writing. In the planning stage, you need to first analyse the question. Pick out the keywords and the issues you have to write about, think of topic sentences and supporting points, decide on a structure (2 main body paragraphs or 3?) then think about how you will write the introduction and a thesis statement.

It sounds like a lot to do but with practice, you can do this in 10 minutes. I have had high-level English students write an IELTS essay with only a minimal 2 minutes of planning and they ended up with a Band 6.

Actually, the most obvious ideas are the best. The IELTS writing exam is not testing your general knowledge or cool ideas.

Let’s take a look at the first essay type for a structure we could use. At this stage you should just be making notes on a topic sentence and supporting points.

Introduction: paraphrase the task question and give a thesis statement

Main body one main idea: If professional people leave their own country it will negatively impact schools, universities and hospitals, so they will lack expertise and experienced staff.



Main body two main idea:   The governments of poorer countries should support professionals with higher salaries, better housing and bonuses to entice them to stay.



Conclusion: Refer to the problem and paraphrase the solution.

Let’s take a look at the second essay type. The essay is the same structure as essay 1 but I have made changes to main body one mainly. The second main body paragraph, which deals with the solutions, is going to be similar to essay 1, but I will also make a point about career advancement chances.

Main body one main idea:   The problem is caused by low salaries and lack of chances for professionals to advance their careers,  also there are better opportunities overseas.

Main body two main idea:   The governments of poorer countries should support professionals with higher salaries and more chances of career advancement to entice them to stay.



Conclusion: Refer to the cause and paraphrase the solution.

Can you match these introductions to the appropriate task questions above?

  • A growing number of qualified personnel, such as doctors and teachers from developing countries, are migrating to work in more developed countries. Due to this, highly qualified people are in short supply, which negatively affects education and health services. To tackle the issue their governments should offer greater financial rewards to entice people to stay.
  • A growing number of qualified personnel, such as doctors and teachers from developing countries, are migrating to work in more developed countries. This is caused by low salaries and a lack of career advancement opportunities. To tackle the issue their governments should offer greater financial rewards and more chances of promotions.

1. A growing number of qualified personnel, such as doctors and teachers from developing countries, are migrating to work in more developed countries. Due to this, highly qualified people are in short supply, which negatively affects education and health services. To tackle the issue their governments should offer greater financial rewards to entice people to stay.

  • It is clear in the thesis statement that I am referring to the problems that happen as a result of these professional people (doctors, teachers, etc) leaving their own developing countries, and a possible way to tackle this.

2. A growing number of qualified personnel, such as doctors and teachers from developing countries, are migrating to work in more developed countries. This is caused by low salaries and a lack of career advancement opportunities. To tackle the issue their governments should offer greater financial rewards and more chances of promotions.

  • In this thesis statement, I am referring to what is causing this issue to happen and a possible way to tackle this.

Any Questions? , leave a comment below

Leave a Comment Cancel reply

Search

  • I nfographics
  • Show AWL words
  • Subscribe to newsletter
  • What is academic writing?
  • Academic Style
  • What is the writing process?
  • Understanding the title
  • Brainstorming
  • Researching
  • First draft
  • Proofreading
  • Report writing
  • Compare & contrast
  • Cause & effect
  • Problem-solution
  • Classification
  • Essay structure
  • Introduction
  • Literature review
  • Book review
  • Research proposal
  • Thesis/dissertation
  • What is cohesion?
  • Cohesion vs coherence
  • Transition signals
  • What are references?
  • In-text citations
  • Reference sections
  • Reporting verbs
  • Band descriptors

Show AWL words on this page.

Levels 1-5:     grey  Levels 6-10:   orange 

Show sorted lists of these words.

 
-->

Any words you don't know? Look them up in the website's built-in dictionary .

Choose a dictionary .  Wordnet  OPTED  both

Problem-solution essays Situation-problem-solution-evaluation

Problem-solution essays are a common essay type, especially for short essays such as subject exams or IELTS . The page gives information on what they are , how to structure this type of essay, and gives an example problem-solution essay on the topic of obesity and fitness levels.

What are problem-solution essays?

Problem-solution

Problem-solution essays consider the problems of a particular situation, and give solutions to those problems. They are in some ways similar to cause and effect essays , especially in terms of structure (see below). Problem-solution essays are actually a sub-type of another type of essay, which has the following four components:

The 'situation' may be included in the essay prompt, in which case it will not be needed in the main body. If it is needed, it can often be included in the introduction, especially for short essays, as with the example essay below . The 'evaluation' may be included as part of the conclusion (also as in the example below), or omitted altogether, especially for short essays. For these reasons, problem-solution essays are more common than situation-problem-solution-evaluation essays (or SPSE essays).

There are two main ways to structure a problem-solution essay. These are similar to the ways to structure cause and effect essays , namely using a block or a chain structure. For the block structure, all of the problems are listed first, and all of the solutions are listed afterwards. For the chain structure, each problem is followed immediately by the solution to that problem. Both types of structure have their merits. The former is generally clearer, especially for shorter essays, while the latter ensures that any solutions you present relate directly to the problems you have given.

The two types of structure, block and chain , are shown in the diagram below. This is for a short essay, which includes the 'situation' in the introduction and 'evaluation' in the conclusion. A longer essay, for example one of around 1,000 words, with citations , would probably have these two sections as separate paragraphs in the main body.





Example essay

Below is a problem-solution essay on the topic of obesity and poor fitness . It uses the block structure . Click on the different areas (in the shaded boxes) to highlight the different structural aspects in this essay, i.e. Situation, Problem, Solution, Evaluation. This will highlight not simply the paragraphs, but also (for problems and solutions) the thesis statement and summary , as these repeat the problems and solutions contained in the main body.

   
         
   
                   

Consumption of processed and convenience foods and our dependence on the car have led to an increase in obesity and reduction in the fitness level of the adult population. In some countries, especially industrialized ones, the number of obese people can amount to one third of the population. This is significant as obesity and poor fitness lead to a decrease in life expectancy , and it is therefore important for individuals and governments to work together to tackle this issue and improve their citizens' diet and fitness. Obesity and poor fitness decrease life expectancy. Overweight people are more likely to have serious illnesses such as diabetes and heart disease, which can result in premature death. It is well known that regular exercise can reduce the risk of heart disease and stroke, which means that those with poor fitness levels are at an increased risk of suffering from those problems. Changes by individuals to their diet and their physical activity can increase life expectancy. There is a reliance today on the consumption of processed foods, which have a high fat and sugar content. By preparing their own foods, and consuming more fruit and vegetables, people could ensure that their diets are healthier and more balanced, which could lead to a reduction in obesity levels. In order to improve fitness levels, people could choose to walk or cycle to work or to the shops rather than taking the car. They could also choose to walk up stairs instead of taking the lift. These simple changes could lead to a significant improvement in fitness levels. Governments could also implement initiatives to improve their citizens' eating and exercise habits. This could be done through education, for example by adding classes to the curriculum about healthy diet and lifestyles. Governments could also do more to encourage their citizens to walk or cycle instead of taking the car, for instance by building more cycle lanes or increasing vehicle taxes. While some might argue that increased taxes are a negative way to solve the problem, it is no different from the high taxes imposed on cigarettes to reduce cigarette consumption. In short, obesity and poor fitness are a significant problem in modern life, leading to lower life expectancy . Individuals and governments can work together to tackle this problem and so improve diet and fitness . Of the solutions suggested, those made by individuals themselves are likely to have more impact, though it is clear that a concerted effort with the government is essential for success. With obesity levels in industrialized and industrializing countries continuing to rise, it is essential that we take action now to deal with this problem.

 
 
           
 

Academic Writing Genres

GET FREE EBOOK

Like the website? Try the books. Enter your email to receive a free sample from Academic Writing Genres .

Below is a checklist for the main body of an essay. Use it to check your own writing, or get a peer (another student) to help you.

The essay is a essay
An appropriate is used, either or
The essay has a clear
Each paragraph has a clear
The essay has strong support (facts, reasons, examples, etc.)
The conclusion includes a of the main points

Next section

Find out about writing definitions and definition essays in the next section.

Previous section

Go back to the previous section about cause & effect essays .

  • Cause/effect

logo

Author: Sheldon Smith    ‖    Last modified: 22 January 2022.

Sheldon Smith is the founder and editor of EAPFoundation.com. He has been teaching English for Academic Purposes since 2004. Find out more about him in the about section and connect with him on Twitter , Facebook and LinkedIn .

Compare & contrast essays examine the similarities of two or more objects, and the differences.

Cause & effect essays consider the reasons (or causes) for something, then discuss the results (or effects).

Discussion essays require you to examine both sides of a situation and to conclude by saying which side you favour.

Problem-solution essays are a sub-type of SPSE essays (Situation, Problem, Solution, Evaluation).

Transition signals are useful in achieving good cohesion and coherence in your writing.

Reporting verbs are used to link your in-text citations to the information cited.

Top Streams

  • Data Science Courses in USA
  • Business Analytics Courses in USA
  • Engineering Courses in USA
  • Tax Courses in USA
  • Healthcare Courses in USA
  • Language Courses in USA
  • Insurance Courses in USA
  • Digital Marketing Courses in USA

Top Specialization

  • Masters in Data Analytics in USA
  • Masters in Mechanical Engineering in USA
  • Masters in Supply Chain Management in USA
  • Masters in Computer Science in USA
  • MBA in Finance in USA
  • Masters in Architecture in USA

Top Universities

  • Cornell University
  • Yale University
  • Princeton University
  • University of California Los Angeles
  • University of Harvard
  • Stanford University
  • Arizona State University
  • Northeastern University
  • Scholarships to Study in USA
  • Project Management Courses in Australia
  • Accounting Courses in Australia
  • Medical Courses in Australia
  • Psychology Courses in Australia
  • Interior Designing Courses in Australia
  • Pharmacy Courses in Australia
  • Social Work Courses in Australia
  • MBA in Australia
  • Masters in Education in Australia
  • Masters in Pharmacy in Australia
  • Masters in Information Technology in Australia
  • BBA in Australia
  • Masters in Teaching in Australia
  • Masters in Psychology in Australia
  • University of Melbourne
  • Deakin University
  • Carnegie Mellon University
  • Monash University
  • University of Sydney
  • University of Queensland
  • RMIT University
  • Macquarie University
  • PR Courses in Australia
  • SOP for Australia Student Visa
  • Data Science Courses in Canada
  • Business Management Courses in Canada
  • Supply Chain Management Courses in Canada
  • Project Management Courses in Canada
  • Business Analytics Courses in Canada
  • Hotel Management Courses in Canada
  • MBA in Canada
  • MS in Canada
  • Masters in Computer Science in Canada
  • Masters in Management in Canada
  • Masters in Psychology in Canada
  • Masters in Education in Canada
  • MBA in Finance in Canada
  • Masters in Business Analytics in Canada
  • University of Toronto
  • University of British Columbia
  • McGill University
  • University of Alberta
  • York University
  • University of Calgary
  • Algoma University
  • University Canada West
  • IELTS requirement for Canada Student Visa
  • Canada Visa Interview
  • Top cities in Canada for International Students
  • Project Management Courses in UK
  • Data Science Courses in UK
  • Public Health Courses in UK
  • Digital Marketing Courses in UK
  • Hotel Management Courses in UK
  • Nursing Courses in UK
  • Medicine Courses in UK
  • Interior Designing Courses in UK
  • Masters in Computer Science in UK
  • Masters in Psychology in UK
  • MBA in Finance in UK
  • MBA in Healthcare Management in UK
  • Masters in Education in UK
  • Masters in Marketing in UK
  • MBA in HR in UK
  • University of Oxford
  • University of Cambridge
  • Coventry University
  • University of East London
  • University of Hertfordshire
  • University of Birmingham
  • Imperial College London
  • University of Glasgow

Top Resources

  • Universities in Germany
  • Study in Germany
  • Masters in Germany
  • Courses in Germany
  • Bachelors in Germany
  • Germany Job Seeker Visa
  • Cost of Living in Germany
  • Best Universities in Germany

Top Courses

  • Masters in Data Science in Germany
  • MS in Computer Science in Germany
  • Marine Engineering in Germany
  • MS Courses in Germany
  • Masters in Psychology in Germany
  • Hotel Management Courses in Germany
  • Masters in Economics in Germany
  • Paramedical Courses in Germany
  • Karlsruhe Institute of Technology
  • University of Bonn
  • University of Freiburg
  • University of Hamburg
  • University of Stuttgart
  • Saarland University
  • Mannheim University
  • MBA in Ireland
  • Phd in Ireland
  • Masters in Computer Science Ireland
  • Cyber Security in Ireland
  • Masters in Data Analytics Ireland
  • Ms in Data Science in Ireland
  • Pharmacy courses in ireland
  • Business Analytics Course in Ireland
  • Universities in Ireland

Study in Ireland

  • Masters in Ireland
  • Courses in Ireland
  • Bachelors in Ireland
  • Cost of Living in Ireland
  • Ireland Student Visa
  • Part Time Jobs in Ireland
  • Trinity College Dublin
  • University College Dublin
  • Dublin City University
  • University of Limerick
  • Dublin Business School
  • Maynooth University
  • University College Cork
  • National College of Ireland

Colleges & Courses

  • Masters in France
  • Phd in France
  • Study Medicine in France
  • Best Universities in Frankfurt
  • Best Architecture Colleges in France
  • ESIGELEC France
  • Study in France for Indian Students
  • Intakes in France
  • SOP for France Visa
  • Study in France from India
  • Reasons to Study in France
  • How to Settle in France

More About France

  • Cost of Living in France
  • France Study Visa
  • Cost of Living in Frankfurt
  • France Scholarship for Indian Students
  • Part Time Jobs in France
  • Stay Back in France After Masters

About Finland

  • Universities in Finland
  • Study in Finland
  • Courses in Finland
  • Bachelor Courses in Finland
  • Masters Courses in Finland
  • Cost of Living in Finland
  • MS in Finland
  • Average Fees in Finland Universities
  • PhD in Finland
  • Jobs in Finland
  • Bachelor Degree in Medicine & Surgery
  • MBBS Courses in Georgia
  • MBBS Courses in Russia
  • Alte University
  • Caucasus University
  • Georgian National University SEU
  • David Tvildiani Medical University
  • Caspian International School Of Medicine
  • Asfendiyarov Kazakh National Medical University
  • Kyrgyz State Medical Academy
  • Cremeia Federal University
  • Bashkir State Medical University
  • Kursk State Medical University
  • Andijan State Medical Institute
  • IELTS Syllabus
  • IELTS Prepration
  • IELTS Eligibility
  • IELTS Test Format
  • IELTS Band Descriptors
  • IELTS Speaking test
  • IELTS Writing Task 1
  • IELTS score validity
  • IELTS Cue Card

IELTS Reading Answers Sample

  • Animal Camouflage
  • Types Of Societies
  • Australia Convict Colonies
  • A Spark A Flint
  • Emigration To The Us
  • The History Of Salt
  • Zoo Conservation Programmes
  • The Robots Are Coming
  • The Development Of Plastic

IELTS Speaking Cue Card Sample

  • Describe A Puzzle You Have Played
  • Describe A Long Walk You Ever Had
  • Describe Your Favourite Movie
  • Describe A Difficult Thing You did
  • Describe A Businessman You Admire
  • Memorable Day in My Life
  • Describe Your Dream House
  • Describe A Bag You Want to Own
  • Describe a Famous Athlete You Know
  • Aquatic Animal

IELTS Essay Sample Sample

  • Best Education System
  • IELTS Opinion Essay
  • Agree or Disagree Essay
  • Problem Solution Essays
  • Essay on Space Exploration
  • Essay On Historical Places
  • Essay Writing Samples
  • Tourism Essay
  • Global Warming Essay
  • GRE Exam Fees
  • GRE Exam Syllabus
  • GRE Exam Eligibility
  • Sections in GRE Exam
  • GRE Exam Benefits
  • GRE Exam Results
  • GRE Cutoff for US Universities
  • GRE Preparation
  • Send GRE scores to Universities

GRE Exam Study Material

  • GRE Verbal Preparation
  • GRE Study Material
  • GRE AWA Essays
  • GRE Sample Issue Essays
  • Stanford University GRE Cutoff
  • Harvard University GRE Cutoff
  • GRE Quantitative Reasoning
  • GRE Verbal Reasoning
  • GRE Reading Comprehension
  • Prepare for GRE in 2 months

Other Resources

  • Documents Required For Gre Exam
  • GRE Exam Duration
  • GRE at Home
  • GRE vs GMAT
  • Improve GRE Verbal Scores

Free GRE Ebooks

  • GRE Preparation Guide (Free PDF)
  • GRE Syllabus (Free PDF)
  • GMAT Eligibility
  • GMAT Syllabus
  • GMAT Exam Dates
  • GMAT Registration
  • GMAT Exam Fees
  • GMAT Sections
  • GMAT Purpose

GMAT Exam Study Material

  • How to prepare for GMAT?
  • GMAT Score Validity
  • GMAT Preparation Books
  • GMAT Preparation
  • GMAT Exam Duration
  • GMAT Score for Harvard
  • GMAT Reading Comprehension
  • GMAT Retake Strategy

Free GMAT Ebooks

  • GMAT Guide PDF
  • Download GMAT Syllabus PDF
  • TOEFL Exam Registration
  • TOEFL Exam Eligibility
  • TOEFL Exam Pattern
  • TOEFL Exam Preparation
  • TOEFL Exam Tips
  • TOEFL Exam Dates
  • Documents for TOEFL Exam
  • TOEFL Exam Fee

TOEFL Exam Study Material

  • TOEFL Preparation Books
  • TOEFL Speaking Section
  • TOEFL Score and Results
  • TOEFL Writing Section
  • TOEFL Reading Section
  • TOEFL Listening Section
  • TOEFL Vocabulary
  • Types of Essays in TOEFL

Free TOEFL Ebooks

  • TOEFL Exam Guide (Free PDF)
  • PTE Exam Dates
  • PTE Exam Syllabus
  • PTE Exam Eligibility Criteria
  • PTE Test Centers in India
  • PTE Exam Pattern
  • PTE Exam Fees
  • PTE Exam Duration
  • PTE Exam Registration

PTE Exam Study Material

  • PTE Exam Preparation
  • PTE Speaking Test
  • PTE Reading Test
  • PTE Listening Test
  • PTE Writing Test
  • PTE Essay Writing
  • PTE exam for Australia

Free PTE Ebooks

  • PTE Syllabus (Free PDF)
  • Duolingo Exam
  • Duolingo Test Eligibility
  • Duolingo Exam Pattern
  • Duolingo Exam Fees
  • Duolingo Test Validity
  • Duolingo Syllabus
  • Duolingo Preparation

Duolingo Exam Study Material

  • Duolingo Exam Dates
  • Duolingo Test Score
  • Duolingo Test Results
  • Duolingo Test Booking

Free Duolingo Ebooks

  • Duolingo Guide (Free PDF)
  • Duolingo Test Pattern (Free PDF)

NEET & MCAT Exam

  • NEET Study Material
  • NEET Preparation
  • MCAT Eligibility
  • MCAT Preparation

SAT & ACT Exam

  • ACT Eligibility
  • ACT Exam Dates
  • SAT Syllabus
  • SAT Exam Pattern
  • SAT Exam Eligibility

USMLE & OET Exam

  • USMLE Syllabus
  • USMLE Preparation
  • USMLE Step 1
  • OET Syllabus
  • OET Eligibility
  • OET Prepration

PLAB & LSAT Exam

  • PLAB Exam Syllabus
  • PLAB Exam Fees
  • LSAT Eligibility
  • LSAT Registration
  • PLAB Accepted Countries
  • TOEIC Result
  • Study Guide

Application Process

  • LOR for Masters
  • SOP Samples for MS
  • LOR for Phd
  • SOP for Internship
  • SOP for Phd
  • Check Visa Status
  • Motivation Letter Format
  • Motivation Letter for Internship
  • F1 Visa Documents Checklist

Career Prospects

  • Popular Courses after Bcom in Abroad
  • Part Time Jobs in Australia
  • Part Time Jobs in USA
  • Salary after MS in Germany
  • Salary after MBA in Canada
  • Average Salary in Singapore
  • Higher Studies after MBA in Abroad
  • Study in Canada after 12th
  • Most Demanding Engineering Fields

Trending Topics

  • Best Education System in World
  • Best Flying Schools in World
  • Top Free Education Countries
  • Best Countries to Migrate from India
  • 1 Year PG Diploma Courses in Canada
  • Germany Post Study Work Visa
  • Post Study Visa in USA
  • Packing List for Indian Students
  • Data Science Vs Data Analytics
  • Public Vs Private Universities in Germany
  • Universities Vs Colleges
  • Difference Between GPA and CGPA
  • Undergraduate Vs Graduate
  • MBA in UK Vs MBA in USA
  • Degree Vs Diploma in Canada
  • IELTS vs TOEFL
  • Duolingo English Test vs. IELTS
  • Canada Vs India
  • Why Study in Canada
  • Cost of Living in Canada
  • Education System in Canada
  • SOP for Canada
  • Summer Intake in Canada
  • Spring Intake in Canada
  • Winter Intake in Canada
  • Accommodation in Canada for Students
  • Average Salary in Canada
  • Fully Funded Scholarships in Canada
  • Why Study in USA
  • Cost of Studying in USA
  • Spring Intake in USA
  • Winter Intake in USA
  • Summer Intake in USA
  • STEM Courses in USA
  • Scholarships for MS in USA
  • Acceptable Study Gap in USA
  • Interesting Facts about USA
  • Free USA course
  • Why Study in UK
  • Cost of Living in UK
  • Cost of Studying in UK
  • Education System in UK
  • Summer Intake in UK
  • Spring Intake in UK
  • Student Visa for UK
  • Accommodation in UK for Students
  • Scholarships in UK
  • Why Study in Germany
  • Cost of Studying in Germany
  • Education System in Germany
  • SOP for Germany
  • Summer Intake in Germany
  • Winter Intake in Germany
  • Study Visa for Germany
  • Accommodation in Germany for Students
  • Free Education in Germany

Country Guides

  • Study in UK
  • Study in Canada
  • Study in USA
  • Study in Australia
  • SOP Samples for Canada Student Visa
  • US F1 Visa Guide for Aspirants

Exams Guides

  • Duolingo Test Pattern

Recommended Reads

  • Fully Funded Masters Guide
  • SOP Samples For Australia
  • Scholarships for Canada
  • Data Science Guide
  • SOP for MS in Computer Science
  • Study Abroad Exams
  • Alumni Connect
  • Booster Program
  • Scholarship
  • Offline Centres

GPA CALCULATOR Convert percentage marks to GPA effortlessly with our calculator!

Expense calculator plan your study abroad expenses with our comprehensive calculator, ielts band calculator estimate your ielts band score with our accurate calculator, education loan calculator discover your eligible loan amount limit with our education calculator, university partner explore growth and opportunities with our university partnership, accommodation discover your perfect study abroad accommodation here, experience-center discover our offline centers for a personalized experience, our offices visit us for expert study abroad counseling..

  • 18002102030
  • Study Abroad

IELTS Problem Solution Essays: Cause And Solution Essay IELTS

  • IELTS Preparation
  • IELTS E-Books
  • IELTS Registration
  • IELTS Exam Fee
  • IELTS Exam Dates 2024
  • Documents Required
  • IELTS Test Centers
  • Test Format
  • Band Descriptors
  • IELTS Speaking Test
  • General Reading Test
  • General Writing Task
  • IELTS Coaching
  • Types of Essays
  • IELTS for Australia
  • IELTS Results
  • Generation Gap Essay
  • GPA Calculator
  • Study Abroad Consultant In India
  • Study Visa Consultants in India

Updated on 26 July, 2024

Sunita Kadian

Sunita Kadian

Ielts expert.

Sunita Kadian

Being an IELTS aspirant myself, I know how challenging the IELTS problem-solution essays can be. These essays require you to identify causes of an issue and propose viable solutions. Mastering this format can help to not only boost your IELTS score but also enhances your critical thinking skills.

So, are you ready to tackle these essays with confidence? Read on to learn the essentials and strategies to excel in the IELTS problem-solution essays.

Table of Contents

Variations of problem-solution questions, tips for answering problem-solution essays, inattentiveness to task instructions, lack of examples, unclear expression of opinion, poor organization and linking of ideas, limited vocabulary and sentence structure, lack of proofreading, sample 1 on ielts problem-solution essay.

  • Sample 2 on problem solution IELTS Essay

Tips to Write Problem-Solution Essay

Problem solution essay structure, popular study abroad destinations.

Discussed below are the variations of the problem-solution questions in IELTS writing task 2:

1. Direct: This problem-solution essay topic explicitly states the problem and asks about solutions.

Sample Questions:

  • The increasing cost of higher education is a barrier to entry for many students. What problems does this create, and how can we offer affordable and quality education for everyone?
  • The rise of social media addiction among teenagers is causing concern. Discuss the negative impacts of social media overuse and propose solutions to encourage responsible use among young people.
  • Many students need help with learning foreign languages. What are the primary reasons for this, and how can the situation be improved?

2. Indirect: This format presents a situation or a trend and asks to discuss its impact (problem) and potential solutions.

  • Automation is increasingly replacing human jobs in many industries. Discuss the challenges automation poses for workers and propose strategies to help people adapt and develop new skills in this changing job market.
  • The growing popularity of urban living has led to overcrowded cities with rising living costs. Discuss the problems associated with urbanization and suggest measures to create more sustainable and affordable cities.
  • The increasing reliance on technology has made face-to-face communication less common. Analyze the impact of this trend on social interactions and suggest ways to encourage stronger interpersonal communication skills.

3. Solution-focused: These are problem-solving essay topics which focus on a specific problem and ask for potential solutions.

  • Traffic congestion in major cities is a major cause of air pollution and wasted time. Propose some effective solutions that could help alleviate traffic problems in urban areas.
  • The growing amount of plastic waste is causing significant environmental damage. Suggest some solutions to reduce plastic consumption and promote sustainable waste management practices.
  • The decline in public transportation ridership in many cities is a concern. Propose strategies to make public transportation more appealing and efficient to encourage people to use it as their primary mode of travel.

4. Cause and Effect: This problem-solution essay in IELTS   presents a problem and asks you to discuss its causes and potential solutions.

  • The gap between rich and poor is widening in many countries. What are the main factors leading to income inequality, and how can we promote policies that create a more equitable society?
  • The spread of misinformation online is a growing problem that undermines trust in media sources. Discuss the causes of the spread of fake news and suggest solutions to promote online literacy and critical thinking skills.
  • Climate change is a global challenge with wide-ranging consequences. Analyze the main causes of climate change and propose solutions to mitigate its impact on our planet.

Here are certain tips for answering problem-solution essays efficiently:

  • Identify the Problem and Solution Clearly: Analyze the question to pinpoint the core problem and any mentioned solutions. Providing a clear definition of the problem and a well-thought-out solution will set the stage for a compelling essay.
  • Develop Strong Arguments: Use clear and concise language, logical reasoning, and transition words to connect your ideas. Building strong arguments that are logically structured and supported by evidence will reinforce the persuasiveness of your essay.
  • Provide Examples: Use real-life examples, statistics, or research to strengthen your arguments for both the problem and the solution. Incorporating relevant examples and credible data will illustrate the significance of the issue and the effectiveness of your proposed solution.
  • Consider Multiple Solutions: If the question allows, discuss alternative solutions and weigh their pros and cons. Exploring various solutions and their implications will demonstrate a comprehensive understanding of the issue and the ability to evaluate different options critically.
  • Don't Just List Problems and Solutions: Choose one or two key issues and develop them thoroughly, explaining their impact and the effectiveness of your proposed solutions. 
  • Focus on Practicality:  Ensure your solutions are realistic and feasible to implement. Emphasizing practical and feasible solutions will portray a sense of realism and understanding of the practical challenges involved in addressing the problem.
  • Link the Problem and Solution: Clearly show how your proposed solution addresses the identified problem. Articulating a clear connection between the problem and the proposed solution will highlight the relevance and effectiveness of your approach.

Common Mistakes to Avoid in IELTS Problem-Solution Essays

Here’s a list of common mistakes you must avoid to incorporate effective essay-writing strategies. This will help you enhance the clarity, coherence, and quality of your IELTS problem-solution essays.

Not reading the task carefully can lead to misunderstanding and incomplete responses.  You can underline keywords for better understanding. For example, in the task "Nowadays many people in cities live alone. What are the reasons for it? Is it a positive or a negative trend?", you must discuss reasons for urban solo living and evaluate its positivity or negativity.

Merely stating points without elaboration or examples weakens your essay. After presenting a reason like career-focused urbanites living alone, you must provide an explanation followed by a concrete example.

Failing to state your opinion in the introduction and conclusion lacks coherence. Make your stance evident throughout the essay to make the reader understand your viewpoint. This clarity is crucial for higher band scores.

Essays lacking clear paragraph structure or logical transitions confuse readers. Each paragraph should have a distinct theme and connect logically to the next using appropriate connectors.

Over-reliance on a narrow vocabulary or repetitive sentence structures can hinder the sophistication of your essay. Aim for lexical variety and diverse sentence structures to demonstrate language proficiency and avoid monotony.

Review your problem-solution essays for spelling, grammar, and punctuation errors to maintain their quality. Spend the last few minutes to check and correct any mistakes to ensure your essay is polished and error-free.

Sample 1 on IELTS Problem Solution Essay

Let us now look at some staples for better understanding of how IELTS problem-solution essays must be dealt with.

A lot of professionals like doctors and engineers are leaving their poor countries to settle down in developed countries. What are the problems? What can be done to control the situation? 

The immigration of talented young citizens negatively affects the economy of the native country. Brain drain has been a serious concern for poor countries. Young IT professionals, doctors, and engineers are looking out for opportunities with the top employers in developed economies of the world. It has even become a matter of pride to leave one’s country to contribute to a foreign economy. As a result, poor countries are getting poorer, and developed ones are reaching new heights. This can be solved by implementing various measures, such as offering competitive pay.

The impact of brain drain is alarming and has caused an economic downturn in poor countries. The conditions created by the phenomenon have led to critical economic degradation and low living standards. It affects not only the economy but also the education and the healthcare sectors. Skills in fields like engineering, medicine, and IT are crucial for maintaining the key economic sectors like food production, business, healthcare, employment, and education in a country. Lack of basic facilities keeps poor countries from developing and encouraging investment.

The best way to tackle and restrict the brain drain from poor countries is to convert  push factors into pull factors. The government needs to create strategies and employment opportunities to increase the retention of talent. Poor countries can put a check on this by offering better work conditions, better pay, and a promising job market. The young generation is likely to stop immigrating if they can access a developed job market with better pay in their native country. This is one way of retaining the skills that the country needs to grow and develop.

Another approach is to develop infrastructure for training and research programs, helping doctors and engineers work in their home countries after completing their courses.

Lastly, better living standards should be offered in order to retain young talents. It is a positive way to inspire the talents to stay back and work for their own economy and develop it.

Another key reason for choosing Canada is the country’s welcoming nature as Canada is a melting pot where people of different ethnicities live together without fear of racial discrimination. Canada greets overseas students in a safe environment and offers the best quality of life and job opportunities.

I would like to conclude by saying that the solutions offered above will be financially demanding, and the poor countries have to look for financial aids to implement these strategies.

Master IELTS for FREE

Unlock your potential with our expertly crafted IELTS FREE Course designed to help you achieve your desired band score. Don't miss this unique opportunity to start your journey towards global success.

In this Course you will find:

  • Expert instructions from experienced instructor.
  • Comprehensive coverage of all IELTS modules.
  • Flexible, self-paced online learning.
  • Interactive lessons and practice tests.
  • Exclusive tips and strategies.

Download E-Books for IELTS Preparation

IELTS IDIOMS GUIDE

Recommended Reads:

Sample 2 on problem solution IELTS Essay 

Many countries do not show enough respect towards the elderly. What are the problems associated with this? List the possible solutions.

Being respectful towards the elderly is not in fashion, it is a fast eroding culture in developed countries, causing serious troubles both on individual and societal levels. Personal priorities are important in modern society but at the cost of what? This is an alarming aspect, but it can be tackled if proper solutions and strategies are adopted. 

Joint families are gradually losing their hold over modern societies. The changing in our lifestyle and daily priorities make it difficult to stay connected. There are a lot of issues emerging due to this change in the approach towards the elderly.

Work patterns and lifestyle are undergoing significant changes. A close look at a corporate worker’s life reveals that they hardly have time for themselves,  let alone others. Work pressure and competition have affected moral values and the race to earn money almost takes away a major portion of the life of the younger generations. After meeting strict deadlines and targets, individuals come home drained both mentally and physically. This need for financial stability has taken a toll on morality in Individuals.

However, what is sad amid all this is the fact that the elderly at home are neglected. They do not have anyone to talk to. Even after having a full-fledged family, they are alienated. Not only this. The health conditions of older generations deteriorate as they do not get adequate care. The number of suicides among the elderly has seen a rise. Some even abandon their homes to live a lonely life.

The elderly generation is suffering from physical abuse and assault as well. Many such cases happen daily in developed economies, and it is depressing. It is also common these days to find elderly homeless seeking shelter in government camps, and their numbers are only rising. The idea of a family seems to be a myth as the older generations, the pillars of the family carrying values and traditions face neglect.

Solutions need to be implemented both at societal and individual levels to address this concern. There should be awareness programs on family values and the roles played by the elderly. Companies should offer work-life balance to employees so that they can have more family time. The elderly deserve attention, care, and respect. The government should implement strict laws and punishment for those that harm the elderly. Lastly, education will play an important role. The school curriculum should also include sessions and programs on elderly care and importance so that the child grows up with the right values. 

As per my opinion, the cycle of life keeps moving and if we are disrespectful today, we will receive the same in the future. Elderly generations are the pillars of the family. We do not have the right to disrespect those that have made uncountable struggles and sacrifices just to bring us up. Elderly concerns need to be dealt with priority.

Related Reads:

  • Word Count and Timing:  Aim for a minimum of 250 words in your Task 2 essay. While there is no maximum word limit, you get only 40 minutes to complete the task, without any extensions.
  • Essay Structure:  Divide your essay into four parts: introduction, problem, solution, and conclusion. The introduction should provide an overview of the essay. The problem section will outline the issues, while the solution section will offer measures to address them. The conclusion should summarize our opinion and the essay.
  • Focus on Problem and Solution:  Address the core questions: what are the problems, and what are the possible solutions? Expand on 2-3 problems and their corresponding solutions, without listing them as bullet points.
  • Stay Relevant:  Ensure your problems and solutions are directly related to the question. For example, if discussing overpopulation, don’t include problems caused by globalization.
  • Keywords:  Identify and focus on keywords in the question.Link problems to solutions clearly for a higher band score.
  • Realistic Solutions:  Offer practical and achievable solutions. Avoid unrealistic or impractical suggestions.
  • Paraphrasing: Increase readability by paraphrasing effectively. This keeps the essay engaging and avoids repetition.
  • Conclusion: The conclusion should not have any new problem or new solution. It should be a summary of all that you have already stated in the essay.  Use concise language to wrap up your essay in 4-5 lines.
  • Use of Idioms:  Use idioms appropriately to enhance your essay, ensuring they are relevant and correctly placed. 
  • Grammar and Proofreading:  Ensure your essay is grammatically correct. Avoid punctuation, spelling, and tenses errors. Proofread in the last 1-2 minutes to catch any mistakes.
  • Sentence Structure: Use short, crisp sentences over lengthy ones for better clarity and impact.
  • Question Analysis: Carefully read the question to understand whether it asks for reasons and solutions or problems and solutions. Answer all parts of the question equally.
  • Tone: Maintain a semi-formal tone. Avoid informal or overly personal language.
  • Practice: Write various problem-solution essays on different topics to build confidence and skill.
  • Use of Facts: Only include valid facts or statistical data if you are certain of their accuracy. Avoid hypothetical numbers.
  • Language Tools: Utilize collocations, linking words, and phrases to make your essay coherent and dynamic.

Problem Solution Essay IELTS topics 

Some topics for task 2 problem-solution essays are provided here to start your practice. Choose any one of the topics and follow the instructions laid out in the article above for better preparation:

Q. Nowadays, many people complain that they are struggling with sleep. What problems can cause insomnia in people? What can be done for a good night's sleep?

Q. Average life expectancy is increasing in developed nations. What problems will it cause for individuals and society? Mention some steps that can be taken to reduce the impact of aging populations.

Q. Stress has become a big problem in many countries across the globe. What are some of the elements in modern society that lead to stress? How can we mitigate stress?

Q. Global warming is one of the grave problems that the world is encountering today. What are the causes of it and what measures can be taken to tackle the issue?

Q. The quality of air and water is declining due to industrial effluents and construction sites. What steps could be taken to mitigate this?

Q. Nowadays, many people are struggling with obesity. What are the factors that contribute to this problem? What are the effects?

By now, you must have understood the problem-solution essays

Here is a reference structure to write an excellent essay. The structure will vary depending on what the question demands., but generally, the essay will have four elements if it requires you to write about the problem, its causes, and its solutions.

Here is the general problem- solution essay structure:

  • Introduction
  • Paraphrase the question
  • Cite one key problem and its cause

2) Main body Paragraph 1 (Problem or Cause)

  • Cite the problem or cause
  • Support the statement with suitable examples 

3) Main body paragraph 2 (Solution)

  • Cite the solution
  • Give details about the problem and write about the remedial measures
  • Provide examples

4) Conclusion

  • Summarize your essay by enlisting all the important information in conclusion.

Tackling  an IELTS problem-solution essay can be challenging, but with the right approach and practice, you can get the hang of it. Focus on clearly linking problems to solutions, and remember to keep your essay well-structured. With the tips mentioned in the article, you’ll be ready to ace your IELTS essay and boost your score!

Study in Canada

Study in Canada & Save up to 20 Lakhs with upGrad Abroad

Study in Australia

Study in Australia & Save up to 20 Lakhs with upGrad Abroad

Study in USA

Study in the USA & Save up to 20 Lakhs with upGrad Abroad

Study in Germany

Study in Germany & Save up to 20 Lakhs with upGrad Abroad

Study in Ireland

Study in Ireland & Save up to 20 Lakhs with upGrad Abroad

study in uk

Study in UK & Save up to 20 Lakhs with upGrad Abroad

How to write a problem-solution essay for IELTS?

Writing a problem-solution essay for the IELTS exam can be challenging, but it can be made easy with a few important key points in mind.This type of essay requires you to write about the problem, its causes, and the solutions. In some cases, only feasible solutions are asked for, so it's crucial to pay attention to the specific demands of the question.

The essay is divided into the following parts:

  • Introduction - Identify the problem and elaborate on the same. It should constitute 60-80 words in length.
  • Body - State reasons for the problem and cite the possible solutions to curb the problem. It should be 100-120 words in length. 
  • Conclusion - In this part, you need to summarize all the points or ideas that you have mentioned in the essay. It should not be more than 60-70 words in length.

What are the instructions for a problem-solution essay?

The instructions for the problem-solution essay can be categorized under the following :

IELTS Problem and Solution Essay : What are the problems related to a particular thing, and how can they be resolved or curbed?

IELTS Cause and Solution  Essay : What is the cause of this problem? What are the remedial measures that can be taken to solve the problem?

IELTS Solution Essay: Cite potent measures that can be taken to solve a particular problem.

Sunita Kadian, co-founder and Academic Head at Yuno Learning is an expert in IELTS and English communication. With a background in competitive exam preparation (IELTS, GMAT, CAT, TOEFL), interview prep, and corporate soft skills training, she has adapted these programs for the Yuno platform. Her dedication and commitment earned high praise from students, especially for her public speaking and advanced speaking English courses. Sunita's mastery of language nuances and articulation makes her a favorite among IELTS students. She holds a Postgraduate degree in English Literature, a B.Ed., and a Post Graduate Diploma in Public Relations.

Important Exams

Important resources for ielts, free study abroad counselling, verify your number for a free session with experts .

By submitting this form, you agree to the Terms of Use and Privacy Policy

Refer Your Friend & Earn upto ₹15000

Help your friend upgrade to a Global Career and earn rewards together.

TRENDING SEARCHES

Ielts sample essay, ielts reading answer.

  • Art is Considered an Important Part of a Society
  • Individual Greed and Selfishness
  • Nowadays the Way Many People Interact
  • Double Question
  • Art and Culture
  • Advantages and Disadvantages of Fast Food
  • Capital Punishment
  • Environmental Problems And Solution
  • Causes And Effects Of Obesity
  • Advantages and Disadvantages of Tourism
  • Government and Society
  • When a Country Develops its Technology
  • Business and Money
  • Prevention is Better than Cure
  • Modern Technology
  • Should Smoking Be Banned In Public Places
  • Easiest Way Of Communication
  • Describe an Expensive Activity that You Enjoy Doing Occasionally
  • Describe an Event You Attended in Which You Didn�t Like the Music Played
  • Describe A Live Sports Match That You Watched
  • Describe A Time When You First Talked in A Foreign Language
  • Describe a Person Who Impressed You in Primary School
  • Describe A Street Market In Your City
  • Describe A Time When You Felt Bored
  • Describe A Time When Your Computer Broke Down
  • Describe A Dinner You Really Enjoyed
  • Describe a Film that Made You Laugh
  • Describe a Time When You Told Your Friend an Important Truth
  • Talk About A Time When You Gave Advice to Someone
  • Describe an Art Exhibition that You Visited
  • Describe A Person Who Wears Unusual Clothes
  • Describe a Difficult Decision that You Once Made
  • Describe a Famous Person You Are Interested In
  • Describe A Time When You Helped A Friend
  • Describe an Occasion When Many People Were Smiling
  • Eco Tourism
  • Mental Gymnastics
  • The Birth Of Scientific English
  • The Psychology Of Innovation
  • Collecting As A Hobby
  • The History Of Glass
  • Crop Growing Skyscrapers
  • Moles Happy As Homes Go Underground
  • The Development Of Museums
  • Biological Control Of Pests
  • Reducing The Effects Of Climate Change
  • This Marvelous Invention
  • The Meaning And Power Of Smell
  • An Introduction To Film Sound
  • The Benefits Of Being Bilingual
  • Green Wave Washes Over Mainstream Shopping
  • The Story Of Silk
  • IELTS Certificate
  • Australia Band Requirements
  • IELTS Full Form
  • IELTS Preparation Books
  • CEFR level IELTS
  • IELTS Validity
  • IELTS score chart

The above tips are the Author's experiences. upGrad does not guarantee scores or admissions.

Call us to clear your doubts at:

  • Grievance Redressal
  • Terms of Use
  • Privacy Policy
  • Report a Vulnerability
  • University Partner
  • Accommodation
  • IELTS Band Calculator
  • Download Study Abroad App
  • Education Loan Calculator
  • upGrad Abroad Office
  • Expense Calculator
  • Knowledge Base
  • Business Partner

Top Destinations

Masters programs.

  • MBA in Germany, IU
  • MIM in Germany, IU
  • MS in CS in Germany, IU
  • MS in Data Analytics in USA, Clark University
  • MS in Project Management in USA, Clark University
  • MS in IT in USA, Clark University
  • MS in Data Analytics & Visualization in USA, Yeshiva University
  • MS in Artificial Intelligence in USA, Yeshiva University
  • MS in Cybersecurity, Yeshiva University

Study Abroad Important Blogs

  • Cost of Study:
  • Cost of Studying in Canada
  • Cost of Studying in Ireland
  • Cost of Studying in Australia
  • Cost of living:
  • Cost of living in UK
  • Cost of living in Australia
  • Cost of living in Germany
  • Cost of living in Ireland
  • Cost of living in Canada
  • Cost of Living in Singapore
  • Cost of Living in Netherlands
  • Career Opportunities:
  • Career Opportunities in Australia
  • Career Opportunities in Germany
  • Job Opportunities in After MS in Canada
  • Job Opportunities After MBA in Australia
  • Job Opportunities After MS in UK
  • IELTS Exam Resources:
  • Academic IELTS
  • IELTS Band Score
  • IELTS Writing Task 2
  • IELTS Slot Booking
  • IELTS Band Score Chart
  • IELTS Score for UK
  • IELTS Score for USA
  • Validity of IELTS Score
  • IELTS Speaking Topics
  • IELTS Reading Tips
  • How to Prepare for IELTS at Home Without Coaching
  • Types of IELTS Exam
  • IELTS Academic vs General
  • IELTS Exam Pattern
  • IELTS Essay
  • IELTS Exam Dates
  • Top Streams:
  • Fashion Designing Courses in Australia
  • Accounting Courses in Canada
  • Management Courses in Canada

The Magoosh logo is the word Magoosh spelled with each letter o replaced with a check mark in a circle.

IELTS Writing Task 2: Causes/Solutions Sample Essay

Woman pointing pencil at head to indicate causes and solutions essay

The “cause and solution” style of IELTS Writing Task 2 question presents a common social problem; your job is to identify the causes of the problem and propose ways the problem could be solved. For more information on this type of Writing Task 2 question, including tips and tricks, you can go to Magoosh’s guide to the different kinds of IELTS Writing Task 2 questions . And here in this post, we’ll go through a full causes and solutions model question and sample response.

The response itself is written at the band 9 level. After you read the essay, there is more explanation as to why this essay gets top marks. And to write a similarly high scored essay, check out our IELTS Writing Task 2 template .

Causes/Solutions Model Essay

This essay is a response to the sample prompt immediately below.

Get a higher IELTS score? Start your online IELTS prep today with Magoosh.

Sample Task

Many large cities around the world lack affordable housing. What problems does a lack of affordable housing cause? How can these problems be overcome? Give reasons for your answer and include any relevant examples from your own knowledge or experience.

Write at least 250 words.

Sample Essay (Band 9)

In almost every part of the world, expensive housing is closely associated with urban life. This is the underlying cause of many different problems, and civic planners are struggling to find solutions. I believe that the biggest, most important problem caused by high housing prices is homelessness. In my opinion, this problem can be addressed through rent control and welfare payments to low-income people.

Expensive urban housing leads to homelessness both directly and indirectly. The housing costs themselves cause some people to lose their homes, but such costs also drive up general prices, which can indirectly cause homelessness. For example, if a grocery store itself pays high rent, it must charge everyone more for food. This in turn forces landlords to charge more for rent in order to feed themselves. Once that happens, their tenants may have trouble paying the higher rent while also paying for more expensive food.

To reduce homelessness and make it easier for people to afford housing, I propose a twofold solution: limits on rent prices and increased welfare payments to the poor. Legally limiting rent fees will make housing much more affordable, while welfare payments can help people deal with the high costs of other goods. To give an example, if the government provides low-income people extra money for food, those people, in turn, have more room in their budget to pay rent. By employing both approaches, rent costs are directly reduced, and costs that compete with rent also go down.

In short, while expensive housing and homelessness are serious problems in cities, regulations and public assistance can help. The right kind of regulations will not only reduce the price of housing but also reduce general cost of living. This kind of government action is a win-win for everyone, whether they are homeless or not.

Word count: 300

Explanation of the Score

This essay meets all of the requirements for Band 9 in Task Achievement, Coherence and Cohesion, Lexical Resource, and Grammatical Range and Accuracy; these are the categories seen in the official IELTS Writing Task 2 band descriptors . To understand this essay’s strengths, carefully read the level 9 descriptors in that document. And to further understand why an essay might receive a band 9, see the detailed score report at the bottom of Magoosh’s sample band 9 discussion essay for IELTS Writing Task 2 .

Additional Model Essays for Task 2

Would you like to see sample essays for all of the most common Writing Task 2 question types on the IELTS? At the links below, Magoosh has you covered!

  • Advantage/Disadvantage Essay
  • Two-Part Question Essay
  • Discussion Essay
  • Agree/Disagree Essay

David Recine

David is a Test Prep Expert for Magoosh TOEFL and IELTS. Additionally, he’s helped students with TOEIC, PET, FCE, BULATS, Eiken, SAT, ACT, GRE, and GMAT. David has a BS from the University of Wisconsin-Eau Claire and an MA from the University of Wisconsin-River Falls. His work at Magoosh has been cited in many scholarly articles , his Master’s Thesis is featured on the Reading with Pictures website, and he’s presented at the WITESOL (link to PDF) and NAFSA conferences. David has taught K-12 ESL in South Korea as well as undergraduate English and MBA-level business English at American universities. He has also trained English teachers in America, Italy, and Peru. Come join David and the Magoosh team on Youtube , Facebook , and Instagram , or connect with him via LinkedIn !

View all posts

More from Magoosh

how to write a problem cause solution essay

2 responses to “IELTS Writing Task 2: Causes/Solutions Sample Essay”

MajidUllah Khan Avatar

Before reading this, i was not able to understand that how do i write advantage and disadvantage essay. After reading this, i came to know, this is a piece of cake.

Magoosh Expert

So glad to hear that the article helped! 😀

Thanks for letting us know, and good luck to you!

Leave a Reply Cancel reply

Your email address will not be published. Required fields are marked *

  • PRO Courses Guides New Tech Help Pro Expert Videos About wikiHow Pro Upgrade Sign In
  • EDIT Edit this Article
  • EXPLORE Tech Help Pro About Us Random Article Quizzes Request a New Article Community Dashboard This Or That Game Happiness Hub Popular Categories Arts and Entertainment Artwork Books Movies Computers and Electronics Computers Phone Skills Technology Hacks Health Men's Health Mental Health Women's Health Relationships Dating Love Relationship Issues Hobbies and Crafts Crafts Drawing Games Education & Communication Communication Skills Personal Development Studying Personal Care and Style Fashion Hair Care Personal Hygiene Youth Personal Care School Stuff Dating All Categories Arts and Entertainment Finance and Business Home and Garden Relationship Quizzes Cars & Other Vehicles Food and Entertaining Personal Care and Style Sports and Fitness Computers and Electronics Health Pets and Animals Travel Education & Communication Hobbies and Crafts Philosophy and Religion Work World Family Life Holidays and Traditions Relationships Youth
  • Browse Articles
  • Learn Something New
  • Quizzes Hot
  • Happiness Hub
  • This Or That Game
  • Train Your Brain
  • Explore More
  • Support wikiHow
  • About wikiHow
  • Log in / Sign up
  • Education and Communications
  • College University and Postgraduate
  • Academic Writing

How to Write a Problem Solution Paper

Last Updated: July 22, 2023 Fact Checked

This article was co-authored by Stephanie Wong Ken, MFA . Stephanie Wong Ken is a writer based in Canada. Stephanie's writing has appeared in Joyland, Catapult, Pithead Chapel, Cosmonaut's Avenue, and other publications. She holds an MFA in Fiction and Creative Writing from Portland State University. This article has been fact-checked, ensuring the accuracy of any cited facts and confirming the authority of its sources. This article has been viewed 150,829 times.

A problem solution paper focuses on a particular problem or set of problems. As the essay writer, you will then need to come up with a solution or several solutions to the stated problem. Problem solution papers are common on exams, as they allow you to explore an issue and use critical thinking to respond with a solution. To write a problem solution paper, start by outlining the paper. Then, follow the structure of a problem solution paper and polish the paper so it is at its best when you turn it in.

Starting the Paper

Step 1 Identify the situation.

  • For example, you may have a main situation like, “obesity and poor fitness,” or “trigger warnings on college campuses.”
  • If you get to choose the situation, make a list of groups you belong to, such as “school,” “family,” “race,” “culture”,” or “local community.” Then, identify a situation or issue you have encountered as a member of one of these groups.

Step 2 Determine the key components of the paper.

  • In the situation component, you will paraphrase the prompt of the paper in your own words.
  • In the problem component, you will state the problem or problems and explain what they are in your own words.
  • In the solution component, you will state your solution or solutions to the problem. You will also explain how it will address the problem.
  • In the evaluation component, you will list the main ideas in the paper and offer a prediction or recommendation based on your solution to the problem.
  • There will only be one situation presented to you in the prompt for the paper. You can then have multiple problems and multiple solutions that link back to the situation.

Step 3 Use the block structure for the outline.

  • Introduction section, where you discuss the situation
  • Transition sentence or paragraph
  • Conclusion section, where you discuss the evaluation

Step 4 Try the chain structure for the outline.

  • Problem 1 and Solution to Problem 1
  • Problem 2 and Solution to Problem 2
  • Problem 3 and Solution to Problem 3

Writing the Paper

Step 1 State the situation in your own words.

  • For example, if the situation in the paper prompt is “obesity and poor fitness,” you may focus on specific aspects of the situation in the introduction. You may look at how the consumption of unhealthy food and the overuse of cars plays into obesity and poor fitness in society.

Step 2 Research the problem or problems.

  • If you cannot find a lot of outside material on the problem, you can collect your own data for the paper. Do this by making a survey that you give to people who are affected by the problem. You can also interview people associated with the problem, or with possible solutions.
  • For example, if you were researching the problem “trigger warnings on college campuses,” you may interview college representatives at your university or college. You may also talk to students on campus.
  • Most problem solution papers written for exams do not require you to cite any outside sources. You may need to cite your sources if you are writing the problem solution paper for a class.

Step 3 Create a strong...

  • For example, if you were writing about the situation “obesity and poor fitness,” you may have the following thesis statement: “Obesity and poor fitness can lead to a decrease in life expectancy, and it is essential that individuals and governments work together to tackle this issue by improving their citizen's diet and fitness.”

Step 4 Identify your solutions.

  • For example, you may come up with a solution that addresses a lack of resources by adding support, money, or more staff. Or you may come up with a solution that addresses the problem by changing an existing practice or habit.

Step 5 Support your solutions with specific examples.

  • For example, if one of your solutions to the problem of obesity and poor fitness is to encourage people to cook at home, you may list a few specific ways people can do this. You may suggest that national eating healthy at home campaign is created, offering recipes online that take less than 30 minutes to prepare at home.

Step 6 Wrap up the paper with an evaluation.

  • For example, you may end up a call to action like, “With rising levels of obesity in our country, it is essential that we take action now to address this serious issue.”

Polishing the Paper

Step 1 Confirm the paper follows a clear structure or outline.

  • You can create a reverse outline using your paper as a guide, where you go through each section and confirm it follows the outline you started with.

Step 2 Check for spelling, grammar, and punctuation.

  • You can also show the paper to a peer, friend, or family member and get them to proofread it for you.

Step 3 Revise the paper to fit the word count.

  • If you are writing the problem solution paper for a class assignment, you may still have a set word count. Check that your paper falls within this word count.
  • ↑ https://www.jccc.edu/student-resources/academic-resource-center/writing-center/files/problem-solution-paper.pdf
  • ↑ http://www.eapfoundation.com/writing/essays/problemsolution/
  • ↑ https://grammar.yourdictionary.com/grammar/writing/how-to-write-a-problem-solution-essay.html
  • ↑ https://www.shsu.edu/centers/academic-success-center/writing/handouts/modes/essays/9.-.Problem.Solution.Essay.pdf
  • ↑ https://clt.library.jwu.edu/c.php?g=1028305&p=7459493
  • ↑ https://clt.library.jwu.edu/c.php?g=1028305&p=7459493#s-lg-box-wrapper-27749528

Community Q&A

Tom De Backer

You Might Also Like

Improve Critical Thinking Skills

About This Article

Stephanie Wong Ken, MFA

A problem solution paper focuses on a particular issue and should include one or more solutions to it. You’ll need to begin the paper by stating the situation in your own words. For example, the situation could be “obesity and core fitness.” Include a thesis statement at the end of your introduction, which could be something like, “Obesity can lead to decreased life expectancy, making it imperative for governments and people to tackle this issue by improving diet and fitness.” The following sections should deal with identifying all of the problems arising from the situation and proposing solutions to them. Try to give examples to explain each solution. For instance, if you say the growth of obesity can be stopped by improving people’s diets, you could propose a national healthy eating campaign. Finally, you should conclude by evaluating the whole paper and making recommendations about how to implement your solutions. For tips from our Writing co-author on how to plan an outline for your problem solving paper, read on! Did this summary help you? Yes No

  • Send fan mail to authors

Reader Success Stories

Hongjuan Yang

Hongjuan Yang

May 18, 2018

Did this article help you?

Hongjuan Yang

Liset Chevarria

Feb 5, 2020

Bertrand Bhikarry

Bertrand Bhikarry

Mar 18, 2020

Cressida Melchik

Cressida Melchik

Sep 25, 2021

Do I Have a Dirty Mind Quiz

Featured Articles

Enjoy Your Preteen Years

Trending Articles

The Office Trivia Quiz

Watch Articles

Make French Fries

  • Terms of Use
  • Privacy Policy
  • Do Not Sell or Share My Info
  • Not Selling Info

Don’t miss out! Sign up for

wikiHow’s newsletter

how to write a problem cause solution essay

Free IELTS lessons signup

home

  • Academic practice
  • General practice
  • Task 1 Academic
  • Task 1 General
  • Task 2 (essay)

IELTS Writing: problem and solution essay

In this lesson you’ll learn how to answer problems/solutions questions in IELTS Writing . This type of questions gives you an issue and asks you to describe some common problems associated with it and propose some possible solutions.

  • See problem-solution question sample
  • Learn how to generate ideas
  • Learn band 9 answering strategies
  • See full band 9 answer

Question sample

This is an example of problem-solution question in IELTS Writing:

Despite a large number of gyms, a sedentary lifestyle is gaining popularity in the contemporary world.

What problems are associated with this?

What solutions can you suggest?

How to answer the question?

Before starting to write your answer, you should think of 1-2 problems and 1-2 solutions, so you know what to write about. In our case:

Problems associated with sedentary lifestyle :

how to write a problem cause solution essay

  • problems with backbone (osteoporosis, scoliosis)

Solutions :

  • promote walking and cycling as safe and attractive alternatives to motorized transport
  • promote visiting gyms and doing exercises

Now, after we’ve generated some ideas for our essay, it’s time to use them in our writing .

Remember : it’s not enough to simply state these facts, you should also extend the ideas in your writing.

Band 9 answer structure

Although there are many possible ways to structure your essay, we’ll use this band 9 answer structure that has been approved by many IELTS examiners:

Band-9 essay structure:

  • Introduction
  • Body paragraph 1 - problems
  • Body paragraph 2 - effects

Let’s take a look at each of these sections in detail:

Introduction Write your introduction in two sentences:

  • Sentence 1 - paraphrase the statement (you can use ‘nowadays/today/these days’ to start):

These days a sedentary lifestyle is becoming more and more popular despite a big number of sport facilities.

  • Sentence 2 - say what you’ll write about in your essay:

This essay will discuss the main problems associated with this epidemic and propose some possible solutions to avoid them.

Body paragraph 1- problems

  • Sentence 1 - summarise the main problems of inactive lifestyle:

The main problems caused by inactive lifestyle are obesity and various spine disorders.

  • Sentences 2-3 - state and explain the first problem (you can also give an example). It’s very important to expand your opinion! Imagine that your examiner doesn’t know this subject at all and you have to explain everything in detail:

A growing number of body research shows that long periods of physical inactivity raise a risk of becoming overweight. This is because people burn fewer calories and easily gain weight.

  • Sentences 4-5 - describe the second problem (as usual, expand your opinion). You can give an example and use linking words ‘ moreover’ , ’ what’s more’ or ‘ also ’ to start:

What’s more, a lot of studies show that so-called ‘sitting disease’ often results in posture and backbone problems. Due to constant sitting, person loses muscle tissue and curves spine, developing numerous spinal diseases. For example, it has been proven that about 80% of people experience backache at least once a week.

Body paragraph 2 - solutions

  • Sentence 1 - briefly state the main solutions:

In my opinion, the best solution to this problem is promoting active lifestyle.

  • Sentences 2-3 - write the first solution and explain it:

Firstly, millions of people stay less active because they use cars instead of walking. Therefore, an effective way to make people more active is to advertise walking and cycling as safe and attractive alternatives to motorized transport.

Moreover, inactive lifestyle is gaining popularity because nowadays a lot of people prefer passive rest to workouts in the gym. And the best way to avoid the hazards of unhealthy living is to obtain a regular dose of physical activity. Thus, promoting gyms and regular exercising would increase the level of activity.

Write your conclusion in 2 sentences by summing up the problems and solutions you’ve written in your body paragraphs:

In conclusion, leading a sedentary lifestyle causes a lot of health problems, including obesity and spinal diseases. The most effective solution is to increase the level of fitness among the society by advertising physical activity.

Band 9 answer sample

These days a sedentary lifestyle is becoming more and more popular despite a big number of sport facilities. This essay will discuss the main problems associated with this epidemic and propose some possible solutions to avoid them.

The main problems caused by inactive lifestyle are obesity and various spine disorders. A growing number of body research shows that long periods of physical inactivity raise a risk of becoming overweight. This is because people burn fewer calories and easily gain weight. What’s more, a lot of studies show that so-called ‘sitting disease’ often results in posture and backbone problems. Due to constant sitting, person loses muscle tissue and curves spine, developing numerous spinal diseases. For example, it has been proven that about 80% of people experience backache at least once a week.

In my opinion, the best solution to these problems is promoting active lifestyle. Firstly, millions of people stay less active because they use cars instead of walking. Therefore, an effective way to make people more active is to advertise walking and cycling as safe and attractive alternatives to motorized transport. Moreover, inactive lifestyle is gaining popularity because nowadays a lot of people prefer passive rest to workouts in the gym. And the best way to avoid the hazards of unhealthy living is to obtain a regular dose of physical activity. Thus, promoting gyms and regular exercising would increase the level of activity.

(268 words)

My Speech Class

Public Speaking Tips & Speech Topics

Problem-Solution Speech [Topics, Outline, Examples]

Photo of author

Jim Peterson has over 20 years experience on speech writing. He wrote over 300 free speech topic ideas and how-to guides for any kind of public speaking and speech writing assignments at My Speech Class.

problem solution

In this article:

Problem-Solution Outline

Problem-solution examples, criminal justice, environment, relationships, teen issues.

What to include in your problem-solution speech or essay?

Problem-solution papers employ a nonfiction text structure, and typically contain the following elements:

Introduction: Introduce the problem and explain why the audience should be concerned about it.

Cause/Effect : Inform the audience on what causes the problem. In some cases, you may also need to take time to dispel common misconceptions people have about the real cause.

Can We Write Your Speech?

Get your audience blown away with help from a professional speechwriter. Free proofreading and copy-editing included.

Thesis Statement: The thesis typically lays out the problem and solution in the form of a question and answer. See examples below.

Solution : Explain the solution clearly and in detail, your problem-solving strategy, and reasons why your solution will work. In this section, be sure to answer common objections, such as “there is a better solution,” “your solution is too costly,” and “there are more important problems to solve.”

Call to Action: Summarize the problem and solution, and paint a picture of what will happen if your final solution is adopted. Also, let the reader know what steps they should take to help solve the problem.

These are the most used methods of developing and arranging:

Problem Solution Method Recommended if you have to argue that there is a social and current issue at stake and you have convince the listeners that you have the best solution. Introduce and provide background information to show what is wrong now.

List the best and ideal conditions and situations. Show the options. Analyze the proper criteria. And present your plan to solve the not wanted situation.

Problem Cause Solution Method Use this pattern for developing and identifying the source and its causes.

Analyze the causes and propose elucidations to the causes.

Problem Cause-Effect Method Use this method to outline the effects of the quandary and what causes it all. Prove the connection between financial, political, social causes and their effects.

Comparative Advantage Method Use this organizational public speaking pattern as recommendation in case everyone knows of the impasse and the different fixes and agrees that something has to be done.

Here are some examples of problems you could write about, with a couple of potential solutions for each one:

Marriage Problem: How do we reduce the divorce rate?

Solution 1: Change the laws to make it more difficult for couples to divorce.

Solution 2: Impose a mandatory waiting period on couples before they can get married.

Environmental Problem: What should we do to reduce the level of carbon dioxide in the atmosphere?

Solution 1: Use renewable energy to fuel your home and vehicles.

Solution 2: Make recycling within local communities mandatory.

Technical Problem: How do we reduce Windows error reporting issues on PCs?

Solution 1: Learn to use dialogue boxes and other command prompt functions to keep your computer system clean.

Solution 2: Disable error reporting by making changes to the registry.

Some of the best problems to write about are those you have personal experience with. Think about your own world; the town you live in, schools you’ve attended, sports you’ve played, places you’ve worked, etc. You may find that you love problem-solution papers if you write them on a topic you identify with. To get your creativity flowing, feel free to browse our comprehensive list of problem-solution essay and paper topics and see if you can find one that interests you.

Problem-Solution Topics for Essays and Papers

  • How do we reduce murder rates in the inner cities?
  • How do we stop police brutality?
  • How do we prevent those who are innocent from receiving the death penalty?
  • How do we deal with the problem of gun violence?
  • How do we stop people from driving while intoxicated?
  • How do we prevent people from texting while driving?
  • How do we stop the growing child trafficking problem?
  • What is the best way to deal with domestic violence?
  • What is the best way to rehabilitate ex-cons?
  • How do we deal with the problem of overcrowded prisons?
  • How do we reduce binge drinking on college campuses?
  • How do we prevent sexual assaults on college campuses?
  • How do we make college tuition affordable?
  • What can students do to get better grades in college?
  • What is the best way for students to effectively balance their classes, studies, work, and social life?
  • What is the best way for college students to deal with a problem roommate?
  • How can college students overcome the problem of being homesick?
  • How can college students manage their finances more effectively?
  • What is the best way for college students to decide on a major?
  • What should be done about the problem of massive student loan debts?
  • How do we solve the global debt crisis?
  • How do we keep countries from employing child labor?
  • How do we reduce long-term unemployment?
  • How do we stop businesses from exploiting consumers?
  • How do we reduce inflation and bring down the cost of living?
  • How do we reduce the home foreclosure rate?
  • What should we do to discourage consumer debt?
  • What is the best way to stimulate economic growth?
  • How do we lower the prime cost of manufacturing raw materials?
  • How can book retailers deal with rising bookseller inventory costs and stay competitive with online sellers?
  • How do we prevent kids from cheating on exams?
  • How do we reduce the illiteracy rate?
  • How do we successfully integrate English as a Second Language (ESL) students into public schools?
  • How do we put an end to the problem of bullying in schools?
  • How do we effectively teach students life management skills?
  • How do we give everyone access to a quality education?
  • How do we develop a system to increase pay for good teachers and get rid of bad ones?
  • How do we teach kids to problem solve?
  • How should schools deal with the problem of disruptive students?
  • What can schools do to improve reading comprehension on standardized test scores?
  • What is the best way to teach sex education in public schools?
  • How do we teach students to recognize a noun clause?
  • How do we teach students the difference between average speed and average velocity?
  • How do we teach math students to use sign charts?
  • How can we make public education more like the Webspiration Classroom?
  • How do we stop pollution in major population centers?
  • How do we reduce the negative effects of climate change?
  • How do we encourage homeowners to lower their room temperature in the winter to reduce energy consumption?
  • What is the best way to preserve our precious natural resources?
  • How do we reduce our dependence on fossil fuels?
  • What is the best way to preserve the endangered wildlife?
  • What is the best way to ensure environmental justice?
  • How can we reduce the use of plastic?
  • How do we make alternative energy affordable?
  • How do we develop a sustainable transportation system?
  • How can we provide quality health care to all our citizens?
  • How do we incentivize people to stop smoking?
  • How do we address the growing doctor shortage?
  • How do we curb the growing obesity epidemic?
  • How do we reduce dependence on prescription drugs?
  • How do we reduce consumption of harmful substances like phosphoric acid and acetic acid?
  • How can we reduce the number of fatal hospital errors?
  • How do we handle the health costs of people living longer?
  • How can we encourage people to live healthier lifestyles?
  • How do we educate consumers on the risk of laxatives like magnesium hydroxide?
  • How do we end political corruption?
  • How do we address the problem of election fraud?
  • What is the best way to deal with rogue nations that threaten our survival?
  • What can our leaders do to bring about world peace?
  • How do we encourage students to become more active in the political process?
  • What can be done to encourage bipartisanship?
  • How can we prevent terrorism?
  • How do we protect individual privacy while keeping the country safe?
  • How can we encourage better candidates to run for office?
  • How do we force politicians to live by the rules they impose on everyone else?
  • What is the best way to get out of a bad relationship?
  • How do we prevent cyberbullying?
  • What is the best solution for depression?
  • How do you find out where you stand in a relationship?
  • What is the best way to help people who make bad life choices?
  • How can we learn to relate to people of different races and cultures?
  • How do we discourage humans from using robots as a substitute for relationships?
  • What is the best way to deal with a long-distance relationship?
  • How do we eliminate stereotypical thinking in relationships?
  • How do you successfully navigate the situation of dating a co-worker?
  • How do we deal with America’s growing drug problem?
  • How do we reduce food waste in restaurants?
  • How do we stop race and gender discrimination?
  • How do we stop animal cruelty?
  • How do we ensure that all citizens earn a livable wage?
  • How do we end sexual harassment in the workplace?
  • How do we deal with the water scarcity problem?
  • How do we effectively control the world’s population?
  • How can we put an end to homelessness?
  • How do we solve the world hunger crisis?
  • How do we address the shortage of parking spaces in downtown areas?
  • How can our cities be made more bike- and pedestrian-friendly?
  • How do we balance the right of free speech and the right not to be abused?
  • How can we encourage people to use public transportation?
  • How do we bring neighborhoods closer together?
  • How can we eliminate steroid use in sports?
  • How do we protect players from serious injuries?
  • What is the best way to motivate young athletes?
  • What can be done to drive interest in local sports?
  • How do players successfully prepare for a big game or match?
  • How should the revenue from professional sports be divided between owners and players?
  • What can be done to improve local sports venues?
  • What can be done to ensure parents and coaches are not pushing kids too hard in sports?
  • How can student athletes maintain high academic standards while playing sports?
  • What can athletes do to stay in shape during the off-season?
  • How do we reduce teen pregnancy?
  • How do we deal with the problem of teen suicide?
  • How do we keep teens from dropping out of high school?
  • How do we train teens to be safer drivers?
  • How do we prevent teens from accessing pornography on the Internet?
  • What is the best way to help teens with divorced parents?
  • How do we discourage teens from playing violent video games?
  • How should parents handle their teens’ cell phone and social media use?
  • How do we prepare teens to be better workers?
  • How do we provide a rational decision-making model for teens?
  • How do we keep companies from mining our private data online and selling it for profit?
  • How do we prevent artificial intelligence robots from taking over society?
  • How do we make high-speed internet accessible in rural areas?
  • How do we stop hackers from breaking into our systems and networks?
  • How do we make digital payments more secure?
  • How do we make self-driving vehicles safer?
  • What is the best way to improve the battery life of mobile devices?
  • How can we store energy gleaned from solar and wind power?
  • What is the best way to deal with information overload?
  • How do we stop computer makers from pre-installing Internet Explorer?

Compare and Contrast Speech [Topics and Examples]

Proposal Speech [Tips + 10 Examples]

1 thought on “Problem-Solution Speech [Topics, Outline, Examples]”

This is very greatfull Thank u I can start doing my essay

Leave a Comment

I accept the Privacy Policy

Reach out to us for sponsorship opportunities

Vivamus integer non suscipit taciti mus etiam at primis tempor sagittis euismod libero facilisi.

© 2024 My Speech Class

ENG1021 Advanced Composition & Communication: Problem-Solution Essay

  • MLA Citations
  • Problem-Solution Essay
  • Evaluating Resources
  • Proofreading

Quick Links

Helpful Databases

  • A-Z Databases Find subject-specific databases related to your topic with A-Z Databases.

Academic Search Premier

  • CQ Researcher
  • EBSCO eBook Academic Collection
  • Emerald Insight
  • Opposing Viewpoints In Context
  • SAGE Journals Online
  • Statistical Abstracts of the United States
  • Taylor & Francis Social Science & Humanities Library

The Research Trajectory/Proposal

Picking a topic.

Think about what problem would you like to see solved. You should pick something that will be motivating to you, something that you are passionate about. Why? For one, it will make it easier for you to research and to keep yourself motivated to do the research. For another, it will make it easier for you to write on and make your paper more engaging for others to read. Your paper and the research it documents should be impactful to those who read it.

Important: make sure that you pick a problem that really has possible solutions. The same considerations apply to picking a topic for this paper as for any other -- the topic can't be too broad or too narrow, and it has to be something that can actually be researched. For more information on choosing a topic, check out this webpage on finding problem topics and useful ways to approach finding solutions to those problems .

Forming research questions

Not sure what to research for your topic? Take the topic you have selected and turn it into a research question. For instance, if your topic is "obesity in America," you could frame that into research questions like "What are measures that prevent weight gain?" or "What are the most sustainable diets for weight loss?" (Tip: Avoid search terms like "best" or "worst" because they are very vague and subjective. Choose terms that are more specific and quantifiable in meaning.)

Still stuck on how to create a research question? Check out this video for more help:

Researching your topic

Now that you have a research question, do some preliminary research to give you direction on breaking your problem down in to feasible solutions. Choose more specific Browse the library's databases, skim through a book, or search the web for possible information.

Keep these questions in mind as you search:

  • What type of information is available on your topic?
  • Where did you find this information?
  • Who are the experts, and what do they have to say about your topic?

Creating a thesis and presenting your proposal

Using the research you gathered from your research question(s), create a claim indicating what solutions you plan to propose in your paper. What three solutions are you proposing to the problem? The topic you submit in your proposal cannot be changed, but the research process is still ongoing, so your solutions may change if you find better ones in your ongoing research. This is a tentative thesis; you can make some changes to it later if you need to.

Now that you have taken some time to seriously think about and research your topic, you are ready to present your trajectory/proposal (intentions) for the research project. The proposal must contain your final subject/topic for the research project.

Not sure how to put together a thesis? Watch this helpful video that breaks it down for you:

Outlining a Problem-Solution Paper

A problem-solution paper is exactly what it sounds like. First, an issue or obstacle is posed, then a solution or series of solutions is suggested to resolve that issue or obstacle.

Problem-solution essays can be structured differently depending on the nature of the topic. (For instance, if there is important historical information to your problem that is a bit too long for your intro, then you might want to spend a paragraph between your intro and your solution body paragraphs to give your audience that history.) Most essays, however, follow this basic formula:

I. Introduction: Communicating the Problem

  • Paint a vivid picture of the problem. Focus on the how and why: how did this become a problem? why is it (still) a problem? why is it important / why does it need to be fixed? 
  • End your intro with a thesis statement recapping the problem and providing a preview of the solution(s) you will offer in the rest of your paper.

II. Body: Communicating the Solution(s)

  • Cover each potential solution to the problem in its own paragraph, or cover each step of a multi-step solution with each step having its own paragraph.
  • Each body paragraph should have a clearly stated claim that is distinct from, though connected to, the other paragraphs.
  • Support each solution or solution step with several pieces of evidence from authoritative sources of information. (Best to go to the library catalog and article databases for these.)
  • Cite each source used to support your claims correctly according to MLA citation format , using an in-text citation that corresponds to an entry in your works cited page.

III. Conclusion: Reiterating Importance

  • Recap your thesis statement (problem + brief overview of solution)
  • Close out your paper by emphasizing the importance of solving the problem you have covered.
  • << Previous: MLA Citations
  • Next: Evaluating Resources >>
  • Last Updated: Nov 8, 2023 5:11 PM
  • URL: https://clt.library.jwu.edu/eng1021

JWU-Charlotte Library:

801 West Trade Street, Charlotte, North Carolina 28202

980 598-1611

  • Location and Directions
  • Off-Campus Access
  • Staff Directory
  • Chat with a Librarian
  • Interlibrary Loan (ILL)
  • System Status
  • Study Rooms
  • Research Appointment
  • Culinary Museum
  • U.S. Locations
  • UMGC Europe
  • Learn Online
  • Find Answers
  • 855-655-8682
  • Current Students

Online Guide to Writing and Research

Thinking strategies and writing patterns, explore more of umgc.

  • Online Guide to Writing

Patterns for Presenting Information

Problem-Cause-Solution Pattern

According to conventional wisdom, you can summarize every story ever told in the following way: someone falls into a hole and must climb out. In other words, every story is about solving a problem. There are probably many exceptions to this observation; however, connecting the need to solve a real-life problem to your subject can draw your readers’ attention. The problem-cause-solution pattern can help you do this.

the word of REVISE on building blocks concept.

In a sense, this pattern is a variety of the specific-to-general pattern, as it often begins with specific details and moves to a somewhat generalized solution. However, rather than evoking a sense of mystery and suspense, the problem-cause-solution pattern focuses on concrete difficulties; and though a solution may appeal to abstract principles, the solution should have a practical application, enough to solve the real-life problem.

When to Use this Pattern

You may find the problem-cause-solution pattern useful in writing case studies, critiques, introductions, reports of scientific investigations, literary reviews, political and social discourse, white papers, proposals, many kinds of reports, and essay examinations.

How to Create this Pattern

The name of the problem-cause-solution pattern also describes the sequence in which to present your information.

Begin by describing the problem.

Proceed through diagnosing and analyzing the problem.

Then propose a solution.

The forms of analysis used to diagnose the problem may vary. You might, for example, use comparative analysis to evaluate for flaws in a process that may have led to the problem. You might use a combination of synthesis and cause and effect analysis to locate systemic conditions which caused the problem. However, in each instance—whether analyzing an entire process or analyzing a specific cause—the goal is to locate a cause or causes.

Example of this Pattern

There are two main kinds of ice that shape sea levels. The first is sea ice, which comes from ocean water that freezes solid. It makes up most of the ice at the North Pole. As it forms, it changes the saltiness of seawater and helps shape powerful ocean currents. 

Melting sea ice doesn’t change the overall amount of water in the ocean, just as melting ice cubes don’t change the water level in a glass of water. But sea ice tends to reflect sunlight, while the darker ocean tends to soak up its heat. That speeds up warming and drives more ice melt in a worrying feedback loop. The warmer temperatures also contribute to the thermal expansion of water, which in turn can raise sea levels. 

The second kind of ice is land ice, which builds up in sheets over thousands of years from compacted snow. In Antarctica, the ice sheet is 1.5 miles thick (2.4 km) on average, reaching up to 3 miles (5 km) in some areas. Greenland’s ice sheet averages a mile in thickness. When land ice starts to jut out over the ocean, it creates a floating ice shelf (Irfan, 2022, paras. 9-11).

Example Explained

Notice how the passage above begins with an implied problem: ice causing changes to sea levels. The passage proceeds to explain the causes of changing sea levels. These are the first two parts of our pattern. A few paragraphs later, the author shifts to discussing the beginnings of a solution.

Key Takeaways

  • The problem-cause-solution approach will first describe the problem, then analyze the cause or responses to the problem, and then will lead to a solution.
  • We practice this approach daily in our interactions with others, whether at work or home.

Mailing Address: 3501 University Blvd. East, Adelphi, MD 20783 This work is licensed under a  Creative Commons Attribution-NonCommercial-ShareAlike 4.0 International License . © 2022 UMGC. All links to external sites were verified at the time of publication. UMGC is not responsible for the validity or integrity of information located at external sites.

Table of Contents: Online Guide to Writing

Chapter 1: College Writing

How Does College Writing Differ from Workplace Writing?

What Is College Writing?

Why So Much Emphasis on Writing?

Chapter 2: The Writing Process

Doing Exploratory Research

Getting from Notes to Your Draft

Introduction

Prewriting - Techniques to Get Started - Mining Your Intuition

Prewriting: Targeting Your Audience

Prewriting: Techniques to Get Started

Prewriting: Understanding Your Assignment

Rewriting: Being Your Own Critic

Rewriting: Creating a Revision Strategy

Rewriting: Getting Feedback

Rewriting: The Final Draft

Techniques to Get Started - Outlining

Techniques to Get Started - Using Systematic Techniques

Thesis Statement and Controlling Idea

Writing: Getting from Notes to Your Draft - Freewriting

Writing: Getting from Notes to Your Draft - Summarizing Your Ideas

Writing: Outlining What You Will Write

Chapter 3: Thinking Strategies

A Word About Style, Voice, and Tone

A Word About Style, Voice, and Tone: Style Through Vocabulary and Diction

Critical Strategies and Writing

Critical Strategies and Writing: Analysis

Critical Strategies and Writing: Evaluation

Critical Strategies and Writing: Persuasion

Critical Strategies and Writing: Synthesis

Developing a Paper Using Strategies

Kinds of Assignments You Will Write

Patterns for Presenting Information: Critiques

Patterns for Presenting Information: Discussing Raw Data

Patterns for Presenting Information: General-to-Specific Pattern

Patterns for Presenting Information: Problem-Cause-Solution Pattern

Patterns for Presenting Information: Specific-to-General Pattern

Patterns for Presenting Information: Summaries and Abstracts

Supporting with Research and Examples

Writing Essay Examinations

Writing Essay Examinations: Make Your Answer Relevant and Complete

Writing Essay Examinations: Organize Thinking Before Writing

Writing Essay Examinations: Read and Understand the Question

Chapter 4: The Research Process

Planning and Writing a Research Paper

Planning and Writing a Research Paper: Ask a Research Question

Planning and Writing a Research Paper: Cite Sources

Planning and Writing a Research Paper: Collect Evidence

Planning and Writing a Research Paper: Decide Your Point of View, or Role, for Your Research

Planning and Writing a Research Paper: Draw Conclusions

Planning and Writing a Research Paper: Find a Topic and Get an Overview

Planning and Writing a Research Paper: Manage Your Resources

Planning and Writing a Research Paper: Outline

Planning and Writing a Research Paper: Survey the Literature

Planning and Writing a Research Paper: Work Your Sources into Your Research Writing

Research Resources: Where Are Research Resources Found? - Human Resources

Research Resources: What Are Research Resources?

Research Resources: Where Are Research Resources Found?

Research Resources: Where Are Research Resources Found? - Electronic Resources

Research Resources: Where Are Research Resources Found? - Print Resources

Structuring the Research Paper: Formal Research Structure

Structuring the Research Paper: Informal Research Structure

The Nature of Research

The Research Assignment: How Should Research Sources Be Evaluated?

The Research Assignment: When Is Research Needed?

The Research Assignment: Why Perform Research?

Chapter 5: Academic Integrity

Academic Integrity

Giving Credit to Sources

Giving Credit to Sources: Copyright Laws

Giving Credit to Sources: Documentation

Giving Credit to Sources: Style Guides

Integrating Sources

Practicing Academic Integrity

Practicing Academic Integrity: Keeping Accurate Records

Practicing Academic Integrity: Managing Source Material

Practicing Academic Integrity: Managing Source Material - Paraphrasing Your Source

Practicing Academic Integrity: Managing Source Material - Quoting Your Source

Practicing Academic Integrity: Managing Source Material - Summarizing Your Sources

Types of Documentation

Types of Documentation: Bibliographies and Source Lists

Types of Documentation: Citing World Wide Web Sources

Types of Documentation: In-Text or Parenthetical Citations

Types of Documentation: In-Text or Parenthetical Citations - APA Style

Types of Documentation: In-Text or Parenthetical Citations - CSE/CBE Style

Types of Documentation: In-Text or Parenthetical Citations - Chicago Style

Types of Documentation: In-Text or Parenthetical Citations - MLA Style

Types of Documentation: Note Citations

Chapter 6: Using Library Resources

Finding Library Resources

Chapter 7: Assessing Your Writing

How Is Writing Graded?

How Is Writing Graded?: A General Assessment Tool

The Draft Stage

The Draft Stage: The First Draft

The Draft Stage: The Revision Process and the Final Draft

The Draft Stage: Using Feedback

The Research Stage

Using Assessment to Improve Your Writing

Chapter 8: Other Frequently Assigned Papers

Reviews and Reaction Papers: Article and Book Reviews

Reviews and Reaction Papers: Reaction Papers

Writing Arguments

Writing Arguments: Adapting the Argument Structure

Writing Arguments: Purposes of Argument

Writing Arguments: References to Consult for Writing Arguments

Writing Arguments: Steps to Writing an Argument - Anticipate Active Opposition

Writing Arguments: Steps to Writing an Argument - Determine Your Organization

Writing Arguments: Steps to Writing an Argument - Develop Your Argument

Writing Arguments: Steps to Writing an Argument - Introduce Your Argument

Writing Arguments: Steps to Writing an Argument - State Your Thesis or Proposition

Writing Arguments: Steps to Writing an Argument - Write Your Conclusion

Writing Arguments: Types of Argument

Appendix A: Books to Help Improve Your Writing

Dictionaries

General Style Manuals

Researching on the Internet

Special Style Manuals

Writing Handbooks

Appendix B: Collaborative Writing and Peer Reviewing

Collaborative Writing: Assignments to Accompany the Group Project

Collaborative Writing: Informal Progress Report

Collaborative Writing: Issues to Resolve

Collaborative Writing: Methodology

Collaborative Writing: Peer Evaluation

Collaborative Writing: Tasks of Collaborative Writing Group Members

Collaborative Writing: Writing Plan

General Introduction

Peer Reviewing

Appendix C: Developing an Improvement Plan

Working with Your Instructor’s Comments and Grades

Appendix D: Writing Plan and Project Schedule

Devising a Writing Project Plan and Schedule

Reviewing Your Plan with Others

By using our website you agree to our use of cookies. Learn more about how we use cookies by reading our  Privacy Policy .

Composition Type: Problem-Solution Essays

  • An Introduction to Punctuation
  • Ph.D., Rhetoric and English, University of Georgia
  • M.A., Modern English and American Literature, University of Leicester
  • B.A., English, State University of New York

In composition , using a problem-solution format is a method for analyzing and writing about a topic by identifying a problem and proposing one or more solutions. A problem-solution essay is a type of argument. "This sort of essay involves argumentation in that the writer seeks to convince the reader to take a particular course of action. In explaining the problem, it may also need to persuade the reader concerning specific causes" (Dave Kemper et al., "Fusion: Integrated Reading and Writing," 2016).

The Thesis Statement

In many types of report writing, the thesis statement is posed front and center, in one sentence. Author Derek Soles writes about how the thesis statement in a problem-solution paper differs from a straight "report of findings" type of text:

"[One]  expository  mode is the problem-solution essay, topics for which are typically framed in the form of questions. Why did fourth-graders from poor families score low on a nationwide math test, and how can educators improve math education for this group? Why is Iran a threat to our national security, and how can we reduce this threat? Why did it take the Democratic Party so long to select a candidate for the 2008 presidential election, and what can the party do to make the process more efficient in the future? These essays have two parts: a full explanation of the nature of the problem, followed by an analysis of solutions and their likelihood of success."
("The Essentials of Academic Writing," 2nd ed. Wadsworth, Cengage, 2010)

Readers need additional context before you get to your thesis, but that is not to say that the thesis has to be posed as a question in the introduction:  

"In a problem-solution essay, the thesis statement usually proposes the solution. Because readers must first understand the problem, the thesis statement usually comes after a description of the problem. The thesis statement does not have to give details about the solution. Instead, it summarizes the solution. It should also lead naturally to the body of the essay, preparing your reader for a discussion of how your solution would work."
(Dorothy Zemach and Lynn Stafford-Yilmaz, "Writers at Work: The Essay." Cambridge University Press, 2008)

Sample Introductions

It can be helpful to see completed examples before writing in order to examine what makes for an effective piece. See how these introductions give some context before posing the topic and lead naturally into the body paragraphs, where the evidence will be listed. You can imagine how the author has organized the rest of the piece.

"We buried my cousin last summer. He was 32 when he hanged himself from a closet coat rack in the throes of alcoholism, the fourth of my blood relatives to die prematurely from this deadly disease. If America issued drinking licenses, those four men—including my father, who died at 54 of liver failure—might be alive today."
(Mike Brake, "Needed: A License to Drink."  Newsweek , March 13, 1994)
"America is suffering from overwork. Too many of us are too busy, trying to squeeze more into each day while having less to show for it. Although our growing time crunch is often portrayed as a personal dilemma, it is, in fact, a major social problem that has reached crisis proportions over the past twenty years."
(Barbara Brandt, "Whole Life Economics: Revaluing Daily Life." New Society, 1995)
"The modern-day apartment dweller is faced with a most annoying problem: paper-thin walls and sound-amplifying ceilings. To live with this problem is to live with the invasion of privacy. There is nothing more distracting than to hear your neighbors' every function. Although the source of the noise cannot be eliminated, the problem can be solved."
(Maria B. Dunn, "One Man's Ceiling Is Another Man's Floor: The Problem of Noise")

Organization

In "Passages: A Writer's Guide, " how to organize a problem-solution paper is explained:  

"Though to some extent [your organization of the paper] depends on your topic, do make sure that you include the following information:
Introduction: Identify the problem in a nutshell. Explain why this is a problem, and mention who should be concerned about it.
Problem Paragraph(s): Explain the problem clearly and specifically. Demonstrate that this is not just a personal complaint, but a genuine problem that affects many people.
"Solution Paragraph(s): Offer a concrete solution to the problem, and explain why this is the best one available. You may want to point out why other possible solutions are inferior to yours. If your solution calls for a series of steps or actions to be followed, present these steps in a logical order.
"Conclusion: Reemphasize the importance of the problem and the value of your solution. Choose a problem that you have experienced and thought about—one that you have solved or are in the process of solving. Then, in the essay itself, you may use your own experience to illustrate the problem. However, don't focus all the attention on yourself and on your troubles. Instead, direct the essay at others who are experiencing a similar problem. In other words, don't write an I essay ('How I Cure the Blues'); write a you essay ('How You Can Cure the Blues')."
(Richard Nordquist, Passages: A Writer's Guide , 3rd ed. St. Martin's Press, 1995)
  • Heuristics in Rhetoric and Composition
  • Learn How to Use Extended Definitions in Essays and Speeches
  • Periodical Essay Definition and Examples
  • Disambiguation in Linguistics and Computational Linguistics
  • Definition and Examples of Science Writing
  • Definition and Examples of Humorous Essays
  • Definition Examples of Collage Essays
  • Using Simple vs Simplistic
  • exploratory essay
  • Pronoun of laziness (grammar)
  • Definition and Examples of Formal Essays
  • Conclusion in Compositions
  • What is Classification in Grammar?
  • Topic In Composition and Speech
  • What Is a Personal Essay (Personal Statement)?
  • Definition and Examples of Riddles

Logo for Montgomery College Pressbooks Network

Want to create or adapt books like this? Learn more about how Pressbooks supports open publishing practices.

Learning Objectives

  • Determine the purpose and structure of cause and effect in writing
  • Understand how to write a cause and effect essay

The Purpose of Cause and Effect in Writing

It is often considered human nature to ask “why?” and “how?” We may want to know how our child got sick so we can better prevent it from happening in the future, or why our colleague received a pay raise because we want one as well. We want to know how much money we will save over the long term if we buy a hybrid car. These examples identify only a few of the relationships we think about in our lives, but each shows the importance of understanding cause and effect.

A cause is something that produces an event or condition; an effect is what results from an event or condition. The purpose of the  cause and effect essay is to determine how various phenomena relate in terms of origins and results. Sometimes the connection between cause and effect is clear, but often determining the exact relationship between the two is very difficult. For example, the following effects of a cold may be easily identifiable: a sore throat, runny nose, and a cough. But determining the cause of the sickness can be far more difficult. A number of causes are possible, and to complicate matters, these possible causes could have combined to cause the sickness. That is, more than one cause may be responsible for any given effect. Therefore, cause and effect discussions are often complicated and frequently lead to debates and arguments.

The Structure of a Cause and Effect Essay

The cause and effect essay opens with a general introduction to the topic, which then leads to a thesis that states the main cause, main effect, or various causes and effects of a condition or event.

The cause and effect essay can be organized in one of the following two primary ways:

  • Start with the cause and then talk about the effects.
  • Start with the effect and then talk about the causes.

For example, if your essay is on childhood obesity, you could start by talking about the effect of childhood obesity and then discuss the cause, or you could start the same essay by talking about the cause of childhood obesity and then move to the effect. Regardless of which structure you choose, be sure to explain each element of the essay completely. Explaining complex relationships requires the full use of evidence, such as scientific studies, expert testimony, statistics, and anecdotes.

Because cause and effect essays determine how phenomena are linked, they make frequent use of words and phrases that denote such linkage. See below Phrases of Causation for examples of such terms.

Phrases of Causation

  • as a result
  • consequently

The conclusion should wrap up the discussion and reinforce the thesis, leaving the reader with a clear understanding of the relationship that was analyzed.

Self-Practice Exercise 4.8

H5P:  Cause and Effect Writing Practice

Exercise Preamble

In this exercise, we will think through the steps of a cause and effect essay. This can be tricky. We’ll start by choosing a topic. You want to pick something you know enough about to make claims about the relationships between cause and effect. For this in-class exercise, there’s no need to do external research, but remember that in a more formal assignment, you’ll want to have good sources for all your claims and to avoid speculation.

Some areas where cause and effect thinking is common include:

  • Health and nutrition

For example, you might write a cause and effect essay about whether violent video games cause children to act out, or whether universal basic income programs cause people to live more stable lives. Choose something you can argue from a position of confidence.

Set a timer and freewrite for five minutes about the topic you have chosen.

Can you identify a potential thesis statement from your freewriting? Remember: The cause and effect essay opens with a general introduction to the topic, which then leads to a thesis that states the main cause, main effect, or various causes and effects of a condition or event. In other words, you don’t need to claim the only cause or effect — we know that ideas are complex.

Organizing Your Draft

Which makes the most sense for what you are trying to argue in your thesis statement?

Using point form, sketch out the structure of your essay: how many paragraphs, what will each one focus on, and how will you support it?

Writing a Cause and Effect Essay

Choose an event or condition that you think has an interesting cause and effect relationship. Introduce your topic in an engaging way. End your introduction with a thesis that states the main cause, the main effect, or both.

Organize your essay by starting with either the cause then effect structure, or the effect then cause structure. Within each section, you should clearly explain and support the causes and effects using a full range of evidence. If you are writing about multiple causes or multiple effects, you may choose to sequence either in order of importance. In other words, order the causes from least to most important (or vice versa), or order the effects from least important to most important (or vice versa).

Use the phrases of causation when trying to forge connections between various events or conditions. This will help organize your ideas and orient the reader. End your essay with a conclusion that summarizes your main points and reinforces your thesis. See  Appendix 1: Readings: Examples of Essays  to read a sample cause and effect essay.

Key Takeaways

  • The purpose of the cause and effect essay is to determine how various phenomena are related.
  • The thesis states what the writer sees as the main cause, main effect, or various causes and effects of a condition or event.
  • The cause and effect essay can be organized in one of these two primary ways:
  • Start with the cause and then talk about the effect.
  • Start with the effect and then talk about the cause.
  • Strong evidence is particularly important in the cause and effect essay due to the complexity of determining connections between phenomena.
  • Phrases of causation are helpful to signal links between various elements in the essay.

Essay 1: Expository essay (15%)

In week 7, you will need to submit an expository essay on one of the following topics. Consider these topics as you work through the rest of this chapter and the next. You will need to choose one topic from one of the rhetorical modes below and write a 750- to 900-word essay. You will need to produce a logically organized essay with a thesis statement, well developed and logically organized paragraphs (with topic sentences), and an introduction and conclusion. You will need to support your ideas using one to three sources and include an APA reference list and citations as outlined in the JIBC APA Style Guide. You need to also demonstrate appropriate use of grammar and correct spelling. Remember, your essay should not just be a story; it should demonstrate logical organization and idea development.

Choose any one of the exercises you have completed for Self-Practice 4.1, 4.2, 4.3, 4.4, 4.5, 4.6, 4.7, 4.8, or 4.9 and develop it into your Expository Essay. Consult with your instructor or your classmates if you are having a hard time deciding which activity to develop further.

You need to submit this assignment to your instructor for marking . (15%)

ENGL Resources Copyright © by Tara Horkoff. All Rights Reserved.

Share This Book

IMAGES

  1. How to Write a Problem Solution Essay: Step-by-Step Instructions

    how to write a problem cause solution essay

  2. How to Write a Problem Solution Essay: Guide with Examples

    how to write a problem cause solution essay

  3. Writing A Problem Solution Essay

    how to write a problem cause solution essay

  4. Write Problem Solution Essay Example

    how to write a problem cause solution essay

  5. How to write an IELTS causes solution essay

    how to write a problem cause solution essay

  6. How to Write a Problem Solution Essay: Guide with Examples

    how to write a problem cause solution essay

VIDEO

  1. ‘Cause/Solution’ essay VS ‘Problem/Solution’ essay

  2. Problem And Solution Essay

  3. Write with Impact: IELTS Essay Topic Breakdown

  4. Problem solution essay 2- Muet guide

  5. How to Write IELTS writing task 2 _ Cause Solution Essay

  6. Free Masterclass on IELTS Problem Solution Essay

COMMENTS

  1. IELTS Problem Solution Essays

    It's not the only possible structure but it's the one I recommend because it's easy to learn and will enable you to quickly plan and write a high-level essay. 1) Introduction. Paraphrase the question. State 1 key problem/cause and related solution. 2) Main body paragraph 1 - Problem or Cause.

  2. How to Write a Problem-Solution Essay: Step-by-Step Instructions

    Step 1: Think about groups that you belong to and the problems that those groups have. Make a list of groups you belong to, such as: Step 2: Make a list of problems you have encountered in some of these groups. Sometimes there is a plan for a solution, but it isn't working, or maybe the plan isn't being enforced.

  3. How To Write a Problem Solution Essay

    Propose practical and reasonable solutions for the first problem. Support each solution with evidence and rationale. Solution 2: Repeat the process for the second problem's solutions. Ensure a clear connection between each problem and its respective solution. Conclusion.

  4. How to Structure a Cause and Solution Essay [IELTS Writing Task 2]

    Thankfully, it is very easy to structure a cause and solution essay for IELTS. You simply need to write four paragraphs, with one body paragraph about the causes and one body paragraph about the solutions: Introduction - introduce the topic. Body paragraph #1 - explain the causes of the problem. Body paragraph #2 - explain the solutions ...

  5. How to write an IELTS causes solution essay

    Step by step guide to writing an IELTS causes solution essay. Updated: January 2024. A common type of IELTS task 2 essay is a problem solution or causes solution essay. Here you will need to write about the causes of the problem in main body one and recommendations or possible solutions that could solve the issue in main body two. I could have ...

  6. An Ultimate Guide to Writing IELTS Problem Solution Essays

    To plan and write a problem solution essay IELTS, you can follow these steps: Understand the question. The first step is to carefully read the question and understand what is being asked. You should identify the problem, the cause (s) of the problem, and the required solution (s). Brainstorm your ideas.

  7. IELTS causes and solutions essay

    In this lesson you will learn how to write a high-scoring causes & solutions essay in IELTS Writing.As an example, the model essay will be about loss of bio-diversity.Basically, causes & solutions essay is one of the most common question types in IELTS writing task 2. This question asks you to describe causes and propose solutions for a given problem.

  8. IELTS Writing Task 2 Problem and Solution Essay Lesson

    Problem: flooding of people's homes and businesses. Solution: build flood barriers or move to higher areas. Problem: loss of agricultural land and starvation. Solution: switch to more suitable crops. Problem: displacement of millions of people. Solution: move people in a planned and orderly way before the floods.

  9. All You Need to Know About Writing a Problem Solution Essay

    These essays cover a pressing issue, examine how it causes problems, and offer solutions to these problems. The topic for problem solution essay papers can be incredibly diverse. The problem could be local, regional, or global. It could affect a wide range of people or be part of the discourse on an arcane and obscure aspect of computing ...

  10. How to Write a Problem-Solution Essay

    1) Take a walk. A good problem-solution paper addresses a problem that is worth pursuing and can be solved practically. World peace is out, sorry. So are your personal gripes with security ...

  11. IELTS causes solutions, problem solutions essays

    Problem solutions and causes solutions essay are very similar but there is a subtle difference. One type asks about the problems and the other type asks about the causes. It is very important to spend a couple of minutes analysing the task question so that you know what to write about in the essay. I have seen many good essays lose marks in ...

  12. Problem-solution essays

    Problem-solution essays consider the problems of a particular situation, and give solutions to those problems. They are in some ways similar to cause and effect essays, especially in terms of structure (see below). Problem-solution essays are actually a sub-type of another type of essay, which has the following four components: Situation.

  13. IELTS Problem Solution Essays: Cause And Solution Essay IELTS

    Discussed below are the variations of the problem-solution questions in IELTS writing task 2: 1. Direct: This problem-solution essay topic explicitly states the problem and asks about solutions. Sample Questions: The increasing cost of higher education is a barrier to entry for many students.

  14. IELTS Writing Task 2: Causes/Solutions Sample Essay

    The "cause and solution" style of IELTS Writing Task 2 question presents a common social problem; your job is to identify the causes of the problem and propose ways the problem could be solved. For more information on this type of Writing Task 2 question, including tips and tricks, you can go to Magoosh's guide to the different kinds of ...

  15. How to Write a Problem Solution Essay

    When wondering how to write a problem solution essay, it's important to describe a problem, convince the reader to care about the problem, and propose a solution. ... Or, explain how the problem evolved from a mundane issue to a worthwhile cause. 3. Outline Your Solution. Once you've drafted your introduction, it's time to dive into the ...

  16. How to Write a Problem Solution Paper: 13 Steps (with Pictures)

    1. Confirm the paper follows a clear structure or outline. Review the paper and confirm it covers the four components of a problem solution paper. Make sure it addresses the problem and the solution in detail. Check that your thesis statement appears in the introduction and in the conclusion sections of the paper.

  17. Problem/solution essay in IELTS writing

    Body paragraph 2 - solutions. Sentence 1 - briefly state the main solutions: In my opinion, the best solution to this problem is promoting active lifestyle. Sentences 2-3 - write the first solution and explain it: Firstly, millions of people stay less active because they use cars instead of walking.

  18. Problem Solving Paper Writing Steps With Sample Essays

    1.Describe the problem and tell why it is important to solve. 2. Explain your solution and why it will work. 3. Refute objections. Problems are easy to see, but finding solutions is not as easy. To write an effective paper, you need to spend some time preparing your ideas. This step-by-step guide leads you through the process of getting your ...

  19. Problem Solution Essay (with Example)

    Problem and solution essays are often used as an assignment at the university level, as well as for many tests such as IELTS and TOEFL. A problem solution es...

  20. Problem-Solution Speech [Topics, Outline, Examples]

    Use these topics as presented to write your problem-solution persuasive essay or research paper, or as a starting point to develop your own topic ideas. In this article: Problem-Solution Outline; ... Problem Cause-Effect Method Use this method to outline the effects of the quandary and what causes it all. Prove the connection between financial ...

  21. Problem-Solution Essay

    Problem-solution essays can be structured differently depending on the nature of the topic. (For instance, if there is important historical information to your problem that is a bit too long for your intro, then you might want to spend a paragraph between your intro and your solution body paragraphs to give your audience that history.)

  22. Patterns for Presenting Information: Problem-Cause-Solution Pattern

    The problem-cause-solution approach will first describe the problem, then analyze the cause or responses to the problem, and then will lead to a solution. We practice this approach daily in our interactions with others, whether at work or home. Mailing Address: 3501 University Blvd. East, Adelphi, MD 20783.

  23. Problem-Solution Essays: Definition and Examples

    A problem-solution essay is a type of argument. "This sort of essay involves argumentation in that the writer seeks to convince the reader to take a particular course of action. In explaining the problem, it may also need to persuade the reader concerning specific causes" (Dave Kemper et al., "Fusion: Integrated Reading and Writing," 2016).

  24. 4.9 Cause and Effect

    The Structure of a Cause and Effect Essay. The cause and effect essay opens with a general introduction to the topic, which then leads to a thesis that states the main cause, main effect, or various causes and effects of a condition or event. The cause and effect essay can be organized in one of the following two primary ways: